Сохранен 527
https://2ch.hk/spc/res/248666.html
Домены arhivach.top и arhivach.site временно не функционируют! Используйте домен ARHIVACH.XYZ.
24 декабря Архивач восстановлен после серьёзной аварии. К сожалению, значительная часть сохранённых изображений и видео была потеряна. Подробности случившегося. Мы призываем всех неравнодушных помочь нам с восстановлением утраченного контента!

Тред тупых вопросов #41

 Аноним 15/01/16 Птн 20:12:30 #1 №248666 
14528779503670.jpg
14528779503681.jpg
14528779503812.jpg
14528779503823.jpg
Тред вопросов о жизни, Вселенной и всем таком.

Спрашиваем то, за что в других местах выдают путёвку в биореактор. Здесь анонимные ученые мирового уровня критически рассмотрят любые гениальные идеи и нарисованные в Paint схемы.

Прошлый тред https://2ch.hk/spc/res/245254.html
Аноним 15/01/16 Птн 20:17:28 #2 №248689 
Дрочат ли астронавты на орбите? Скорее всего да, а потому вопрос: им вылают резинки, чтобы семя не разлеталось по станции? Или может резиновые вагины какие-то.
Очевидно, что каким-то образом они сбрасывают напряжение, кроме того, долго обходиться без эякуляции - опасно для здоровья.
Аноним 15/01/16 Птн 20:18:12 #3 №248694 
>>248689
Да, есть специальные презики. В треде космофактов была про это статья.
Аноним 15/01/16 Птн 20:18:31 #4 №248696 
>>248694
Носом не ткнёшь?
Аноним 15/01/16 Птн 20:19:11 #5 №248700 
>>248696
Нет, тред очень большой и он уплыл уже походу. Неделю в него не заходил. сори, анончик.
Аноним 15/01/16 Птн 20:20:21 #6 №248706 
>>248700
А ты случайно не путаешь с теми, что в мочеприёмниках (история с размерами)?
Аноним 15/01/16 Птн 20:27:12 #7 №248719 
>>248689
>>248694
Лолчто? Космонавты в космосе дрочат, и все об этом знают, кэк!
Аноним 15/01/16 Птн 20:28:13 #8 №248724 
>>248719
Да я и говорю, читал где-то, но не могу вспомнить. Толи космофактов тред, толи где ещё. Вроде в том треде было.
Аноним 15/01/16 Птн 20:31:53 #9 №248739 
>>248719
Было бы удивительно, если бы они не дрочили.
Вот про говно сортир столько инфы, а про дрочку - нихуя. Что за хуйня?
Аноним 15/01/16 Птн 20:35:06 #10 №248749 
А сперма потом перерабатывается в воду, да?
Аноним 15/01/16 Птн 20:37:49 #11 №248758 
>>248749
Разлагается на белки, прессуется и белковыми кубиками подаётся на стол.
Аноним 15/01/16 Птн 21:28:39 #12 №248853 
Хочу отправить на орбиту мелкоспутник 200г, умею хорошо делать сорбитовую карамельку, взлетит или нет? Как ракету расчитать?

УИ движка на карамельке около 120 реальный.
Аноним 15/01/16 Птн 21:31:35 #13 №248855 
>>248853
Начни с формулы Циолковского.
Аноним 15/01/16 Птн 21:33:15 #14 №248857 
>>248855
ну я щитал для 5 ступеней получается что-то ебическое под 15 тонн.
Аноним 15/01/16 Птн 21:34:03 #15 №248858 
>>248857
Ну вот. Ты ответил на свой вопрос.
Аноним 15/01/16 Птн 21:35:51 #16 №248860 
>>248858
Хуева...
Надо тогда попробовать карамельку раскачать хотя бы до 200, вроде бы около 400кг получается, хотя бы реально уже сделать.
Аноним 15/01/16 Птн 21:36:15 #17 №248861 
>>248860
хотя 200 нереально
Аноним 15/01/16 Птн 21:36:25 #18 №248862 
>>248853
Бюджет какой? может ты Маск?
Аноним 15/01/16 Птн 21:44:55 #19 №248876 
>>248862
Пока на стадии оценки вообще выполнимости задачи.

Калийка где-то 50-60кр/тонна, сорбит примерно 150кр/тонна, т.е. если уложиться в 300-400кг то около 100-150кр.

Аноним 15/01/16 Птн 21:46:59 #20 №248880 
>>248876
На тему сахара http://www.popmech.ru/technologies/7489-v-kosmos-na-sakhare-karamelnoe-toplivo/#full

Там народ сотни тысяч долларов спускает на карамельные ракеты для прыжка за 100 км.

https://ru.wikipedia.org/wiki/Ламбда-4S пока что самая маленькая ракета.
Аноним 15/01/16 Птн 21:48:51 #21 №248882 
>>248876
> 300-400кг
Можешь даже не пробовать.
Аноним 15/01/16 Птн 21:49:10 #22 №248883 
>>248880
Не особо понимаю куда можно спустить сотни тысяч долларов, если карамель пиздец дешевая.

Ну 150 конечно спизданул, потому что раза три на земле тесты провести да замерить графички тяги да точнее оценить уи с тягой, итого под 400-500к.

Аноним 15/01/16 Птн 21:56:57 #23 №248888 
Так ладно карамель дерьмо.

Где битард может купить звуковую ракету?
Аноним 15/01/16 Птн 21:57:45 #24 №248890 
>>248883
Управление, разделение ступеней, вот это вот все.

Та же лямбда, как приемер:
>Первые три ступени ракеты не имели никаких средств управления, и полностью полагались на пассивные аэродинамические стабилизаторы. Ракета стартовала поэтому с наклонной рампы. Перед отделением четвёртой ступени от третьей микродвигатели на перекиси водорода разворачивали продольную ось ракеты параллельно оси вращения единственного гироскопа, хранившего требуемое направление вектора тяги последней ступени. После этого связка из пустой третьей ступени, четвёртой и спутника раскручивалась вокруг продольной оси, для поддержания нужной ориентации, третья ступень с системой управления отделялась, а четвёртая сообщала спутнику необходимую скорость.
Аноним 15/01/16 Птн 21:58:10 #25 №248891 
14528842903290.jpg
>>248888

Аноним 15/01/16 Птн 21:59:42 #26 №248892 
>>248890
Сейчас есть легкие гироскопы или полностью че нить вроде ардупайлот, нихуя не весит рулит по пограмме как надо, можно лучше намного все сделать
Аноним 15/01/16 Птн 22:00:35 #27 №248894 
>>248892
ну под гироскопами имеется в виду твердотельные акселерометры, я хз как работает, пьеза какая-то по колебаниям работает прецессию регит или еще чета, хуй знает главное что пашет.
Аноним 15/01/16 Птн 22:02:57 #28 №248896 
>>248892
>гироскопы или полностью че нить вроде ардупайлот

Вот эти насиловали ардуину с пруфами http://spacelin.ru/novosti/ocherednoe-ispytanie-sistemy-upravleniya-v-polete/ но потом отказались http://spacelin.ru/novosti/letayushchiy-stend-sletal-v-pyatyy-raz/
Аноним 15/01/16 Птн 22:06:00 #29 №248898 
>>248896
Чет ссылки не те вставил, вот http://spacelin.ru/novosti/prototip-sistemy-upravleniya/
http://spacelin.ru/novosti/pervaya-raketa-lin-indastrial-podgotovka-k-pusku/
Аноним 15/01/16 Птн 22:08:32 #30 №248899 
>>248898
Тяги длинной в пиздарики см, ну и какой-то самодел тупой (4 бита охуеть), проще взять готовый ардупайлот котороый на квадрики ставят и все уже искаропки пашет.
Аноним 15/01/16 Птн 22:12:15 #31 №248903 
Shugar shot to space гугли, но они вроде заглохли.
Аноним 15/01/16 Птн 22:13:19 #32 №248905 
>>248903
Да похер не вывести на карамельке уже посчитал. УИ 100-120 делает ракету 20-30 тонн.
Аноним 15/01/16 Птн 22:15:43 #33 №248907 
>>248899
>уже искаропки пашет
Ну и пихло бери готовое тогда, например http://dzerzhinsky.all.biz/dvigateli-raketnye-tverdotoplivnye-g871619
Аноним 15/01/16 Птн 22:16:41 #34 №248910 
>>248905
Спизди от ЗРК ракету. Уверен, в рушке можно найти прапора, который за бутылку ящик водки тебе отдаст никому не нужный дилдак от какого-нибудь С-75. Выкинешь лишнее, и запустишь в космос свой гопро.
Аноним 15/01/16 Птн 22:17:40 #35 №248912 
>>248910

Я хочу запустить туда локон волос своей мертвой тянки.
Аноним 15/01/16 Птн 22:17:52 #36 №248913 
>>248910
Круто будет, бля. Сосачер на орбите. Прямая трансляция их его спизженой сычевальи спешите на стрим!
Аноним 15/01/16 Птн 22:19:14 #37 №248914 
>>248912
Крематорий дешевле.

>>248913
Для орбиты надо уже МБР доставать, или даже пару, и городить вязанку.
Аноним 15/01/16 Птн 22:19:58 #38 №248915 
Так, ладно, ракета херня.

А если с каким-нибудь крупным спутником отправить? Сколько стоит килограммовую хуйню выкинуть из ракеты?
Аноним 15/01/16 Птн 22:20:59 #39 №248916 
>>248913
Это будет акт протеста против недоступности космоса для ширнармасс. Жертвенный акт, во имя всего человечества.

>>248915
За сто штук вполне можно пульнуть кубик.
Аноним 15/01/16 Птн 22:21:54 #40 №248917 
>>248912
http://naked-science.ru/article/sci/space-burial
Аноним 15/01/16 Птн 22:22:30 #41 №248918 
>>248916
стотыщ бачей? чет дохуя как-то.
Аноним 15/01/16 Птн 22:23:17 #42 №248920 
>>248918
А ты думал. Это только всякие максы могут миллиарды хз где брать на такое.
Аноним 15/01/16 Птн 22:23:41 #43 №248921 
>>248918
А хули ты хотел? С обедов не отложишь и у мамки не спиздиш. Молись на Муска.
Аноним 15/01/16 Птн 22:27:28 #44 №248924 
>>248921
Я думал кубсат 2-3к стоит.
а тут на нахуй 50к пиздс
Аноним 15/01/16 Птн 22:28:54 #45 №248925 
>>248913
ШОК! СЕНСАЦИЯ! СОСАЧЕР РАЗВЕНЧИВАЕТ МИФЫ О КОСМОСЕ И НЕВЕСОМОСТИ! ПОДПИСКА НА СТРИМ 4,49 В МЕСЯЦ! НЕ УПУСТИ!
Аноним 15/01/16 Птн 22:29:55 #46 №248926 
>>248925
Жаль, крауд не взлетит.
Аноним 16/01/16 Суб 02:34:33 #47 №248988 
14529008731660.jpg
Это всё, конечно весело, но что там у буржуев? Ссылки не нужны, просто пара предложений: что там с американским сверхтяжем и ITER?

"Пилят/пилят" не канает.
Аноним 16/01/16 Суб 03:50:05 #48 №248993 
Где там каталину наблюдать, вроде в районе медведицы нихуя не видно. Как она хоть выглядеть должна, может из-за освещения не видно ?
Аноним 16/01/16 Суб 06:10:26 #49 №249000 
>>248993
>Где там каталину наблюдать
>может из-за освещения не видно ?

1. Жить поближе к северу.
2. Жить в медвежьем углу, где темно.
Аноним 16/01/16 Суб 09:13:14 #50 №249012 
>>249000
То есть уральскому быдло не посмотреть?
мимо
Аноним 16/01/16 Суб 15:38:51 #51 №249080 DELETED
Чому не сделать 10-и ступенчатую водяную ракету и морской старт
Аноним 16/01/16 Суб 15:56:51 #52 №249088 
14529490116730.jpg
14529490116871.jpg
Первая фотка - галактика андромеды, выделил черным прямоугольником часть, которую буду увеличивать.
Вторая фотка - та самая увеличенная часть.
Вопрос если эта яркая точка - звезда, то что это за фигня вокруг? Маленькие бесчисленные звезды? Или пыль или вообще что?
Аноним 16/01/16 Суб 16:11:17 #53 №249093 
>>249088
Так твоя звезда-то не из галактики Андромеды же. А из Млечного Пути скорее всего.
Аноним 16/01/16 Суб 16:51:38 #54 №249121 
>>249088
Это шум матрицы фотоаппарата на высоких ISO.

Аноним 16/01/16 Суб 17:14:20 #55 №249136 
>>249088
Большая белая клякса - это звезда, которая находится в нашем Млечном пути, а вот мелкая крупа на втором плане- звезды уже Андромеды. Нельзя сфотографировать просто исключительно Андромеду, в ее сторону полно звезд нашей галактики. Пыль - это вот эти темные разводы на первой фотке.
Аноним 16/01/16 Суб 17:55:22 #56 №249158 
>>249136
Так где пыль?
Аноним 16/01/16 Суб 18:03:38 #57 №249164 
14529566184910.jpg
>>249158
Везде блядь
Аноним 16/01/16 Суб 18:23:58 #58 №249169 
Допустим, сложная жизнь повсеместно распространена в нашей Галактике. Допустим также, образовалась разумная цивилизация на планете, что вращается вокруг Сверх массивной чёрной дыры по типу планеты-океана в фильме "Интерстеллар". В результате время для них течёт, как и в фильме, в шестьдесят тысяч раз раз медленнее, чем для остальных - один час для их цивилизации равен семи годам для всех остальных цивилизаций.
Вопрос: будет ли эта цивилизация, живущая на орбите СМЧД, иметь какие-то преимущества перед остальными или наоборот будет безумно запоздалым по развитию даун-шифтером и никому ненужной?
Аноним 16/01/16 Суб 18:29:27 #59 №249170 
>>249169
Ну, например, в сравнении с нами они будут уберменшами, жрущими обогащенный уран на завтрак. Выход дальше орбиты в космос будет сильно затруднен. А так да, будут себе отставать в развитии.
Аноним 16/01/16 Суб 18:39:24 #60 №249173 
14529587642560.webm
Зачем Билл решил зашквариться о сервис цифровой дистрибуции игр от ЕА?
Аноним 16/01/16 Суб 18:42:40 #61 №249174 
>>249173
>Зачем
За денежкой.
Аноним 16/01/16 Суб 18:43:00 #62 №249175 
>>249173
Хорошая попытка, пиарщик ЕА, но у нас этой штуки все равно нет.
[SIDJEI] Аноним 16/01/16 Суб 19:05:09 #63 №249178 
>>248666 (OP)
пацаны, у кого есть сайт где типа указаны орбиты всех спутников в ирл тайме.
полтора года назад был, но после починки ноута всё стёрлось.
невкрысу
Аноним 16/01/16 Суб 19:17:19 #64 №249183 DELETED
>>249178
ok.ru
[SIDJEI] Аноним 16/01/16 Суб 19:27:11 #65 №249190 DELETED
>>249183
>ok.ru
ХУЙ СОСИ ТУПОЙ ДВАЧЕР, ОБОССАЛА
Аноним 16/01/16 Суб 20:14:17 #66 №249205 DELETED
>>249190
>ОБОССАЛА
Нарушаете гражданка? Быстро, решительно сиськи в тред. То что это спйсач, не дает права нарушать правила.
Аноним 16/01/16 Суб 20:33:13 #67 №249214 
>>248689
чего, какая еще опасность для здоровья
Аноним 16/01/16 Суб 20:52:59 #68 №249222 
>>249214
Лол. Щас тебе эта Марьиванна раскажет всю правду.
Аноним 16/01/16 Суб 22:05:58 #69 №249234 
>>248666 (OP)
На Марсе вообще есть какие-то полезные ископаемые? Или источники энергии, которым потенциальные колонии смогут себя обеспечить?
Аноним 16/01/16 Суб 22:06:34 #70 №249235 
>>249234
>источники энергии, которым потенциальные колонии смогут себя обеспечить?
Солнечная.
Аноним 16/01/16 Суб 22:08:16 #71 №249236 
>>249235
И все? Из чего у него грунт? Один песок? А под грунтом что?
Аноним 16/01/16 Суб 22:28:17 #72 №249239 
>>249169
>Допустим также, образовалась разумная цивилизация на планете, что вращается вокруг Сверх массивной чёрной дыры по типу планеты-океана в фильме "Интерстеллар".
Если даже взять очень маломассивный Юпитер, и крайне близкий к нему спутник Ио, то хуйня получается. Орбита спутника лежит всего в семи радиусах от центра планеты, а пидорасит его - спаси господи.

Ощутимое замедление времени будет только у горизонта. И вряд ли там возможны планеты, не говоря уж о жизни.
Аноним 16/01/16 Суб 22:33:49 #73 №249240 
>>249169
Зачем тебе это знать? Эти знания не пригодятся тебе в этой жизни.
Аноним 16/01/16 Суб 22:34:41 #74 №249241 
>>249239
>>249169
Маловероятны не значит невозможны. Чёрт с ней, с жизнью, но вот замедления времени действительно не будет.
Аноним 16/01/16 Суб 22:36:03 #75 №249242 
>>249236
>Из чего у него грунт? Один песок?
https://ru.wikipedia.org/wiki/Марс#.D0.93.D1.80.D1.83.D0.BD.D1.82
>А под грунтом что?
https://ru.wikipedia.org/wiki/Марс#.D0.93.D0.B5.D0.BE.D0.BB.D0.BE.D0.B3.D0.B8.D1.8F_.D0.B8_.D0.B2.D0.BD.D1.83.D1.82.D1.80.D0.B5.D0.BD.D0.BD.D0.B5.D0.B5_.D1.81.D1.82.D1.80.D0.BE.D0.B5.D0.BD.D0.B8.D0.B5
Аноним 16/01/16 Суб 22:42:17 #76 №249245 
>>249241
>Маловероятны не значит невозможны
Если так подходить - то да. И все молекулы воздуха могут собраться в одной половине комнаты.

>но вот замедления времени действительно не будет.
Ну, вроде как будет. ККС подтверждает, да и время замедляется не только на релятивистских скоростях, но и около больших масс (знаменитая эквивалентность).
Аноним 16/01/16 Суб 22:50:24 #77 №249246 
>>249245
>И все молекулы воздуха могут собраться в одной половине комнаты.
Вы табличку "сарказм" потеряли.

>Ну, вроде как будет.
Хорошо, но не так как в фильме точно.
Аноним 16/01/16 Суб 23:11:25 #78 №249248 
>>249246
>Вы табличку "сарказм" потеряли.
А это не сарказм. Знаменитый пример, который приводят классе в седьмом. Из-за того, что тепловое движение молекул хаотично, ничто не мешает им принять любую конфигурацию (ограниченную, например, комнатой). Полный рандом, вероятность которого, правда, крайне низка. Как и существования цельного массивного тела очень близко к горизонту/эргосфере.

>Хорошо, но не так как в фильме точно.
Так я и пишу, что фильм - параша. Правда, не смотрел его (но осуждаю). Хотя на харкаче уже узнал содержание в деталях.
Аноним 16/01/16 Суб 23:17:40 #79 №249250 
>>249248
>А это не сарказм
Каюсь, туплю.

>Так я и пишу, что фильм - параша.
Фильм эмоционально силён - музыка, атмосфера, игра актёров, кажущаяся реалистичность происходящего из-за обилия умных слов. Погружаешься. Если смотреть технические детали - да, фильм просто художка и искать там физику несколько глупо.
Гравитация - то же самое, только ещё хуже. Я ржал прямо в кинотеатре.
Аноним 16/01/16 Суб 23:33:37 #80 №249255 
>>249250
>кажущаяся реалистичность происходящего из-за обилия умных слов
Не завсегдатаям этой доски.
>Гравитация - то же самое, только ещё хуже
По сравнению с интерстоляром просто эталон правдоподобия. И давай закончим этот несоответствующий теме разговор.
Аноним 16/01/16 Суб 23:35:07 #81 №249256 
>>249255
Поддвачну насчёт гравитации.
мимо
Аноним 17/01/16 Вск 00:01:26 #82 №249261 
>>249256
>Гравитация
А я не поддвачну. Я и его осуждаю, хоть и его не смотрел.

Там ведь, вроде, Сандра БезБулок снималась? Кто такого дубаря на выход в открытый космос взял? Да и остальные - дубари. Тут соседний тред есть, так там чувак всякие реальные случаи описывает. Космонавты, такое ощущение, не зассут (иногда, в прямом смысле) в любой ситуации: это просто "анти-двощеры". А тупняк при выходе в открытый космос - это вообще пиздец. Паника, жопа, что они там еще сделали (ебанули все спутники и станции?). Глупость и паника, вроде так? Или что, смотреть дорогущие декорации и нахуй не нужные "эмоции"? Я, для этого, лучше "Трою" или "Звездные войны" посмотрю.
Аноним 17/01/16 Вск 00:03:23 #83 №249262 
>>249164
А разве это не тамошняя пыль разного цвета?
Аноним 17/01/16 Вск 00:05:11 #84 №249263 DELETED
Платина:
Были ли американцы на Луне?
Аноним 17/01/16 Вск 00:08:04 #85 №249264 DELETED
>>249263
Нет, они были на Венере.
Аноним 17/01/16 Вск 00:08:34 #86 №249265 DELETED
>>249264
Пруфы?
Аноним 17/01/16 Вск 00:12:09 #87 №249267 DELETED
>>249265
госдеп Венера близнец Земли
Аноним 17/01/16 Вск 00:15:47 #88 №249270 
>>249261
Ты странный какой-то, не смотрел, сам в курсе, что не очень хорошо знаешь фильм, но при этом критикуешь. Нет, я понимаю что это модно, но можно не здесь?
Аноним 17/01/16 Вск 00:17:21 #89 №249272 DELETED
>>249270
>можно не здесь?
Мы, как бы, закончили уже
>>249255-кун

>>249263
Были. Вопросы?
Аноним 17/01/16 Вск 00:18:28 #90 №249273 
>>249270
Знаешь, я вот этот кун >>249256
Но в его словах есть доля правды. Просто гравитация на фоне остальных боливудских высеровдействительно смотрится весьма реалистично, другое дело, что таких людей в космос точно не отравят. это он верно подметил. Это ведь не за хлебом сходить, там выдержка, подготовка и прочее на высочайшем уровне. А так да.
sageАноним 17/01/16 Вск 00:22:34 #91 №249275 
>>249266 (OP)
ни мона, а нуна, сюда бля>>248666 (OP)
Аноним 17/01/16 Вск 00:22:53 #92 №249276 DELETED
>>249272
Почему больше не летают а если летали, то накуя столько экспедиций? Просто это больше похоже на многосерийное шоу, нежели на научную работу и как они умудрились выжить в радиации?
Аноним 17/01/16 Вск 00:23:07 #93 №249277 
>>249275
хм не получилось, счас сделаем дубль
Аноним 17/01/16 Вск 00:23:31 #94 №249278 DELETED
>>249276
Не были они там, успокойся.
Аноним 17/01/16 Вск 00:24:37 #95 №249281 DELETED
>>249278
Я ещё даже не начинал.
Аноним 17/01/16 Вск 00:26:34 #96 №249288 
>>249136
>>249136
А могут ли эти "кляксы" быть звездами Андромеды, но просто более яркими чем вся остальная крупа звезд?
Аноним 17/01/16 Вск 00:31:14 #97 №249292 
>>249288
Нет. Предел звездной массы не позволяет звездам достигнуть такой светимости.
Аноним 17/01/16 Вск 00:33:59 #98 №249294 
>>249288
Нет, не могут.
Аноним 17/01/16 Вск 01:31:05 #99 №249337 
14529834654340.jpg
Спейсаны, добра всем, не хочу в мув идти ради такого банального вопроса. Тут увлекающихся больше кто то наверняка видел.

Потерял серию фильмов из 4 частей по моему, познавательный, там молодой (лет 40) харизматичный ведущий (англичанин) рассказывает о зарождении и развитии вселенной. Ну и там по делены на части, допустим целая часть про элементы и их зарождение, Обалденный видеоряд, медитативное повествование, раньше на сон ставил, найти не могу, винт погорел. Подскажет кто, а?? Вещь хорошая.
Аноним 17/01/16 Вск 02:03:21 #100 №249357 
>>249240
Ты что, дурак?
Аноним 17/01/16 Вск 03:15:18 #101 №249390 
>>249337
Ну вспомнил кто? Задолбали какашками в друг друга кидаться.
Аноним 17/01/16 Вск 03:20:41 #102 №249392 
>>249390
Год какой?
Аноним 17/01/16 Вск 03:39:40 #103 №249400 
>>249262
Естественно это пыль Андромеды, я именно это имел ввиду, просто не подумал, что это не очевидно.
Аноним 17/01/16 Вск 03:43:56 #104 №249403 
>>249337
>там молодой (лет 40) харизматичный ведущий (англичанин)
Брайан Кокс подходит.
Wonders of the Universe или The Big Bang Machine
Или в Педивикии список есть.
Аноним 17/01/16 Вск 03:52:25 #105 №249407 
>>249242
я отмечу что для луны больше данных чем для марса, что это за описание грунта, писок, гидрооксид железия 15! процентов.
Что за чушь, я должен по этим данным предположить что марс состоит из железия на 7 процентов ?
Это же бред.
Аноним 17/01/16 Вск 03:54:32 #106 №249408 
>>248666 (OP)
Солнечная Система движется со скоростью 828,000 км/ч вокруг центра Галактики.
Но я немого найти инфу с какой скоростью наша Галактика движется в пространстве, ведь если дальние Галактики удаляются от нас почти со скоростью света, значит и мы летим через Вселенную со всей нашей Галактикой с аналогичной скоростью?
Аноним 17/01/16 Вск 03:54:41 #107 №249409 
>>249407
Это данные для грунта, а не для планеты в целом. Железа в Марсе должно быть намного больше.
Аноним 17/01/16 Вск 03:55:23 #108 №249410 
>>249408
>Но я немого найти инфу с какой скоростью наша Галактика движется в пространстве
Относительно чего?
Аноним 17/01/16 Вск 03:55:37 #109 №249411 
14529921376240.jpg
>>249337
Может этот? тоже Брайн
Аноним 17/01/16 Вск 04:00:49 #110 №249412 
>>249411
У Грина, насколько мне известно, только The Fabric of the Cosmos на PBS был. И вроде на русский не переводился.
Алсо, Грин американец.
Аноним 17/01/16 Вск 04:02:31 #111 №249413 
>>249412
https://www.youtube.com/watch?v=rFZVsWK0xCA
Аноним 17/01/16 Вск 04:09:00 #112 №249415 
>>249413
Про этот не знал.
Аноним 17/01/16 Вск 04:18:58 #113 №249420 
>>249409
это была гипербола, и речь конечно о коре марса, и даже собственно о верхнем слое это коры.
ползают марсоходы, задачей которых собственно и является исследование грунта, и в вике нет нормальной статьи на тему марсианского грунта, что за хуйня.
Справедливости ради, следует отметить, что попадалось получше описание, но не на много, не существенно лучше.
Аноним 17/01/16 Вск 04:29:09 #114 №249421 
14529941500120.jpg
>>249420
Это был какой-то неуместный бугурт. Не нравится статья в вики - перепиши, или переведи английскую, коих, кстати аж три. Марсианская почва, строение Марса и геология Марса. Инфы хуева тьма с ссылками и прочей требухой. Не хочешь википедию - гугли.
Аноним 17/01/16 Вск 12:19:20 #115 №249464 
Как вырваться из гравитационного колодца?
Аноним 17/01/16 Вск 12:53:47 #116 №249485 
Почему на на Плутоне есть криовулканы? Это же самый холодный углок в солнечной системе и там нет приливных взаимодействий.
Аноним 17/01/16 Вск 13:00:46 #117 №249489 
>>249485
Пиздец, тут Харон недавно открыли вообще-то
Аноним 17/01/16 Вск 13:03:52 #118 №249490 
14530250327760.gif
>>249421
это свежий бугурт просто, и походу по какойто причине я не посмотрел английскую версию (что обычно я делаю в первую очередь)
Но да спасибо что подметил, в данном случае я несколько не справедливо наехал на вики.

Аноним 17/01/16 Вск 13:05:57 #119 №249492 
>>249464
в едином трудовом порыве, в смычке города и деревни, программеров и технологов, индусов и китайцев.
Аноним 17/01/16 Вск 13:19:57 #120 №249494 
14530259972010.jpg
14530259972101.jpg
>>248666 (OP)
Космач, поясни, я просто упорот или действительно вселенная нас толсто троллит?

Абсолютно черное тело (чд), свет может попадать только в нее, при этом она попердывает релятевисткими лучами, хуй пойми что из себя представляющие, но нечто совершенно отличное от света.

Зрачек - абсолютно черный, только получает и не отражает свет и позволяет нам видеть преобразовывая волны света в сигналы для мозга.


Внешнее сходство этих СОВЕРШЕННО разных обьектов просто поражает меня, и внезапно можно говорить о том что ЧД может быть расцена как "наблюдатель" квантовых процессов. Я не могу более точно сформулировать, но бля неужели ничего нет на эту тему. Я искал - не нашел, но бля у меня и формулировки даже нет.
Аноним 17/01/16 Вск 13:21:01 #121 №249495 
>>249494
БЛЯЯЯЯЯЯЯЯДЬ ТЫ ШТО ЛОХ ЧТОЛИ ПИЗДЕС
Аноним 17/01/16 Вск 13:27:19 #122 №249496 
>>249494
Ну ты ебанутый
Аноним 17/01/16 Вск 13:30:08 #123 №249497 
>>249494
Боженька так задумал.
Аноним 17/01/16 Вск 13:30:37 #124 №249498 
>>249494
Всё правильно. Поддерживаю автора поста. Сам задумался. Вполне может быть, что это гигантские видеокамеры-глаза вселенной, без шуток.
Аноним 17/01/16 Вск 13:32:11 #125 №249499 
>>249498
>>249494
Семён, будьте бдительны.
Аноним 17/01/16 Вск 13:45:40 #126 №249509 
14530275410510.jpg
>>248666 (OP)
>В 1990 и 1991 годах в космос полетели первые коммерческие космонавты Тоёхиро Акияма (Япония) и Хелен Шарман (Великобритания), которые совершили полёты на советскую орбитальную станцию «Мир» на космических кораблях Союз ТМ-11/Союз ТМ-10 и Союз ТМ-12/Союз ТМ-11 по частно-финансируемым негосударственным проектам телекомпании TBS и «Джуно» (консорциум британских компаний).

А сколько это стоило, билет на Мир, не могу найти конкретную информацию?

https://ru.wikipedia.org/wiki/Акияма,_Тоёхиро
https://ru.wikipedia.org/wiki/Шармен,_Хелен
https://en.wikipedia.org/wiki/Project_Juno
Аноним 17/01/16 Вск 13:54:58 #127 №249513 
14530280986080.jpg
>>249499
Чини детектор.

>>249498
А Хокинговское излучение из чд, имеет какие-то особые квантовые эффекты, я не настолько хорошо разбираюсь в квантовой механике, но там был нюанс что на горизонте рождается пара частица-античастица.
При этом я ничего больше не могу сказать, я даже вывод из этого никакой сделать не берусь. Но в этом что-то есть.

Спейсач, я же знаю, тут есть бородатые седовласые мудрецы которые в уме радиус Шварцшильда на глаз прикинуть могут. Может есть мысли по поводу этого сходства?
[SIDJEI] Аноним 17/01/16 Вск 13:56:08 #128 №249515 
>>249494
>>249498
ТО ЧУВСТВО КОГДА НАРКОМАНЫ АРТВОРК ЗА ПОЛОЖЕНИЕ В ВЕШЕЙ В ИРЛ ПРИНИМАЮТ
Аноним 17/01/16 Вск 14:09:16 #129 №249517 
14530289569670.gif
>>249515
Че ты там кукарекнул? Зрачек не черный? Черная дыра на фоне свечения галактики не создает из-за релятивистского линзирования не создает гало которое четко выделяет само черное тело и "корону". Что позволяет нашему черному зрачку увидеть то место где нет излучения, наш инструмент (глаз) детектирует отсутствие сигнала и мы можем заглянуть в бездну. Ну а классик сказал, что бездна ничем не хуже и невозбранно пялится на нас.
Аноним 17/01/16 Вск 14:14:51 #130 №249519 
>>249517
>ЗРАЧОК НЕ ЧОРНИЙ7?????
Нет. Он далеко не абсолютно чёрный, в отличие от ЧД - любой предмет отражает свет, кроме ЧД.

Пиздец, во что превратился /spc.
Аноним 17/01/16 Вск 14:27:12 #131 №249523 
14530300321430.jpg
14530300321431.jpg
>>249519
Ты тупой что ли? Перввый пик Планковское черное тело, второй пик глаз.
Зрачок блядь, это как диафрагма фотоаппарата - дырка. Свет туда попадает и не может быть отражен наружу уже никак.
Короче иди читай учебник по физике, ибо ты не понимаешь что значит "черное".
Аноним 17/01/16 Вск 14:38:53 #132 №249527 
>>249523
Кто нибудь дайте ребёнку фотку кота, что-ли. Детей лучше наглядностью, им ещё рано логикой объяснять.
Аноним 17/01/16 Вск 14:40:31 #133 №249529 
>>249523
>Зрачок блядь, это как диафрагма фотоаппарата - дырка. Свет туда попадает и не может быть отражен наружу уже никак.
Поссал на тупого ублюдка, не могущего в разграничение математических моделей от реальности. "Планковское чёрное тело" - модель, и, цитируя Википедию (https://ru.wikipedia.org/wiki/%D0%90%D0%B1%D1%81%D0%BE%D0%BB%D1%8E%D1%82%D0%BD%D0%BE_%D1%87%D1%91%D1%80%D0%BD%D0%BE%D0%B5_%D1%82%D0%B5%D0%BB%D0%BE): "Абсолютно чёрных тел в природе не существует, поэтому в физике для экспериментов используется модель", "При нагревании этой полости у неё появится собственное видимое излучение.".
Кстати, сегодня воскресение. В церкви был? Так и представляю, как тебе церковник хуй в рот засунул и наступило озарение: А ВЕДЬ ЧОРНИЕ ДИРИ ЕТО КАК ГЛАЗА!!!!
Аноним 17/01/16 Вск 14:41:17 #134 №249530 
>>249529
https://ru.wikipedia.org/wiki/%D0%90%D0%B1%D1%81%D0%BE%D0%BB%D1%8E%D1%82%D0%BD%D0%BE_%D1%87%D1%91%D1%80%D0%BD%D0%BE%D0%B5_%D1%82%D0%B5%D0%BB%D0%BE
Аноним 17/01/16 Вск 14:55:23 #135 №249537 
1
Аноним 17/01/16 Вск 15:00:14 #136 №249541 
>>249527
Я изначально говорил про человеческий глаз, с первого же поста.
>>249529
"абсолютно черное тело" как и "черная дыра" просто название. Принцип абсолютно точно описывает глаз и черную дыру.
> "При нагревании этой полости у неё появится собственное видимое излучение."
Фокусировка изображения обеспечивается за счёт изменения кривизны хрусталика, которая регулируется цилиарной мышцей. При увеличении кривизны хрусталик становится более выпуклым и сильнее преломляет свет, настраиваясь на видение близко расположенных объектов.
Мы конечно не видим как орлы, часть света отражается внутри глаза и поглощается не в само оптимальной точке на глазном дне. Но ты сука не понимаешь даже то, что ты сам тупо копируешь блядь. КОНЕЧНО ГЛАЗ СОЗДАЕТ ВНУТРИ СЕБЯ СВЕЧЕНИЕ. Исследование глазного дна как раз об этом - когда смотрят почему внутренние отражения в глазу не происходят так-как должны.
Короче просто уходи от сюда нахуй ты блядь конченый.

Аноним 17/01/16 Вск 15:18:37 #137 №249549 
>>249541
Я бы лучше сравнил ЧД и твой анус. В него тоже залетаю разные вещи, прямо как в ЧД. А потом оттуда брызжет тугая струя поноса, это типа джеты.
Аноним 17/01/16 Вск 15:18:46 #138 №249550 
14530331268660.jpg
14530331268681.jpg
>>249541
>Я изначально говорил про человеческий глаз, с первого же поста.
Ну вот что за непослушный ребёнок. Говорят же тебе взрослые, а ты всё споришь.
[SIDJEI] Аноним 17/01/16 Вск 15:25:35 #139 №249557 
>>249523
БЛЯ, SATAN CONFIRMED
[SIDJEI] Аноним 17/01/16 Вск 16:02:22 #140 №249566 
>>249178
ну же, там типа этого, но с графоном получше и спутников в раз 40 больше
http://science.nasa.gov/iSat/?group=SMD
Аноним 17/01/16 Вск 16:45:27 #141 №249574 
Есть ли более-менее вменяемые пруфы о космонавтах до Гагарина?
Аноним 17/01/16 Вск 16:47:48 #142 №249575 
>>249574
И откуда вы, блядь, лезете? Вчера Луна, сегодня Гагарин. В /zog съеби, зелёный
Нет. Только домыслы
Аноним 17/01/16 Вск 16:52:25 #143 №249581 
>>249575
Но ведь Оберт говорил что в 58 был неудачный старт
Аноним 17/01/16 Вск 16:55:44 #144 №249584 
>>249581
Пруфы или не говорил.
Аноним 17/01/16 Вск 17:01:57 #145 №249587 
>>249584
http://www.astronaut.ru/bookcase/article/article61.htm?reload_coolmenus
Аноним 17/01/16 Вск 17:05:58 #146 №249589 
>>249587
Я как-то немцам больше верю. В 1944 уже был суборбитальный полёт.
Аноним 17/01/16 Вск 22:48:57 #147 №250355 
>>249513
>А Хокинговское излучение из чд, имеет какие-то особые квантовые эффекты
1. Это лишь гипотеза, что оно вообще есть.
2. О каких эффектах ты говоришь?
Аноним 17/01/16 Вск 22:56:24 #148 №250384 
>>249574
>пруфы о космонавтах до Гагарина?
Да и о Гагарине нет, не ссы. Все космонавты - военные летчики, у них секретность - ебись конём.
Аноним 18/01/16 Пнд 00:43:19 #149 №250560 
Если кратеры на луне от метеоритов, то где эти метеориты?
Аноним 18/01/16 Пнд 00:49:08 #150 №250566 
>>250560
Не от метеоритов. Они все слишком похожи внешне. Это что-то другое. Может, это остатки от ядерных взрывов большой мощности? Там случилась ядерная война и всё. Вследствие неё погибли все живые организмы, а из-за мощных взрывов атмосферу и всё, что грубо говоря не прикручено было к поверхности луны улетело в космос.
Аноним 18/01/16 Пнд 00:55:35 #151 №250573 
>>250560
оличный вопрос, кадет, на луне.
sageАноним 18/01/16 Пнд 01:30:52 #152 №250598 
>>249587
>Но если Оберт был весьма осторожен в своих высказываниях, подчеркнув, что знает о космической катастрофе с чужих слов и не может ручаться за правдивость этой информации, то агентство Континенталь выдавало на-гора одну сенсацию за другой.

Следующий!
>>249589
>Я как-то немцам больше верю
Раздел религии в другом месте. Никого не ебет во что ты там веришь. Либо факты, либо нахуй.

Аноним 18/01/16 Пнд 07:50:36 #153 №250677 
>>250598
Факт - немцы сделали суборбиту 70 с хуем лет назад.
В СССР тоже что-то было, но видимио решили не вытаскивать трупы на свет. Ещё вопрос с тем, летал ли Гагарин не снят.
Аноним 18/01/16 Пнд 07:52:22 #154 №250678 
>>250677
Видео же есть.
Аноним 18/01/16 Пнд 08:21:46 #155 №250689 DELETED
>>250678
У пиндосав тоже видео есть, и фотографий много. И что?
Аноним 18/01/16 Пнд 08:26:49 #156 №250693 DELETED
>>250689
У нас реальное, у них кубрик снял
Аноним 18/01/16 Пнд 08:27:30 #157 №250694 DELETED
>>250693
Кубрик же у наших спиздил идеи насчёт невесомости.
Аноним 18/01/16 Пнд 08:53:34 #158 №250702 
>>250677
>Факт - немцы сделали суборбиту 70 с хуем лет назад.
Без пилотов.
Аноним 18/01/16 Пнд 08:54:29 #159 №250704 
>>250702
Ну и что. Факт в том. что смогли. И мозги у них были для этого, причём одни из самых лучших.
Аноним 18/01/16 Пнд 09:34:32 #160 №250715 
>>250704
Что "ну и что"? Вопрос был, напомню:
"Есть ли более-менее вменяемые пруфы о космонавтах до Гагарина?".
Никаких данных о нацистах в космосе, либо уровня IronSky.
Аноним 18/01/16 Пнд 10:47:05 #161 №250746 
>>248666 (OP)
Правда что у космонавтов повышенная вероятность получение катаркты? Читал что пидорашкинский космонавт(с усами) чуть ли не половину уже ослеп.

Еще та же не стенки а аллюминиевая хуета в 5 см, как они там не дохнут от радиации?
Аноним 18/01/16 Пнд 10:48:54 #162 №250747 
>>248666 (OP)
Я так и не нашел ни одного доказательства Черной Дыры. Да нашли какие-то СТРАННЫЕ гравитационные величины, но больше нихуя.

Ткни носом наукач, если есть хоть что-то стоящее по этой теме.
Аноним 18/01/16 Пнд 10:53:01 #163 №250749 
>>248666 (OP)
Читал кто-то Салахутдинова Гелий - Блеск и нищета К. Э. Циолковского? Я пытался осилить, но первые 25% книги какие-то извинения, извинения, сравнения с мушкетерами. Короче не осилил, вкратце, что там с циликом белками и стрелками?
Аноним 18/01/16 Пнд 11:20:27 #164 №250770 
>>250747
Нет никаких Доказательств и Святых Откровений.
Совершенно точно можно сказать только одно: есть очень массивные и компактные объекты. Их называют черными дырами. Что там внутри них и как и из чего они состоят - большой вопрос, на который наука пытается дать ответ. Как может и как умеет. И науке сейчас очень тяжело это делать, так как она не может наблюдать их напрямую, да еще не имеет при этом внятной теории квантовой гравитации.
Аноним 18/01/16 Пнд 14:53:39 #165 №250837 
14531180190920.png
14531180190931.png
Сколько бы New Shepard мог вытянуть на НОО, если добавить вторую ступень?
Аноним 18/01/16 Пнд 15:00:08 #166 №250841 DELETED
>>250837
Килограмм 100 или около того.
Аноним 18/01/16 Пнд 15:04:46 #167 №250844 
>>250837
> 5т вторая ступень, с 1.2км/с вверх до первой космической вбок
Вряд ли бы она даже саму себя вытянула.
Аноним 18/01/16 Пнд 15:53:04 #168 №250864 
>>250841
>>250844
А если еще и третью добавить? Что-то типа такого можно сотворить https://en.wikipedia.org/wiki/Black_Arrow ?
Аноним 18/01/16 Пнд 19:49:53 #169 №250971 
>>249408
>ведь если дальние Галактики удаляются от нас почти со скоростью света, значит и мы летим через Вселенную со всей нашей Галактикой с аналогичной скоростью?
Да, если они от нас удаляются, то и мы от них удаляемся.

>>250746
>как они там не дохнут от радиации?
Там не так много, около 1 миллизиверта в день. Это примерно в 5 раз больше чем при полёте обычным авиарейсом. Риск развития болячек повышается но не критично. Кстати при межпланетном полёте до Марса по датчикам Куриосити облучение будет ещё в 2 раза сильнее.
В общем риск есть но долететь вполне можно. Три года полёта на Марс как 6 лет на орбите в МКС.
Аноним 18/01/16 Пнд 19:54:14 #170 №250974 
>>250971
Вдогонку про риски. Говорят, накопленная радиация в 1 Зиверт повышает риск раковых заболеваний на 5%. Таким образом 3 года полёта к Марсу при дозе в полёте 2 миллизиверта в день - повысит риск раковых заболеваний на 10%

Цифры конечно грубые но чтобы масштаб понять пойдут.
Аноним 18/01/16 Пнд 23:44:07 #171 №251042 
Я не хочу умирать.
Аноним 19/01/16 Втр 01:59:42 #172 №251123 
>>250971
в 350 раз выше фона, на земле, многовасто чето так.
Они там профилактики какие делают относительно этого или нет ?
Аноним 19/01/16 Втр 03:38:03 #173 №251142 
Где я могу просвящаться по астрономии преимущественно? Приличная энциклопедия стоит 2к, и в той всего 200 страниц. В библиотеку ходить трудно, ибо в городе не прописан, а чужим на руки не выдают, только там сиди и залупу чиши. Ну и представьте сколько времени займёт путь туда\обратно и самое главное просвящение. А я может покурить выхожу часто. Не выгода по времени, я только к 40 стану адептом
В интернете охуительные интернет магазины, где предлагают почитать д е м о н с т р а ц и ю, или заказать. Но я с таким же успехом могу пойти и купить энциклопедию о которой писал выше.
Аноним 19/01/16 Втр 03:52:41 #174 №251145 
>>251142
>2016
>бумажные источники информации
Аноним 19/01/16 Втр 04:40:47 #175 №251148 
Мы знаем, что рано или поздно нашей сраной Земляшке пиздец. И это не страшно, т.к. всегда можно съебать на другую планету.

Но также рано или поздно придет пиздец галактике и вообще вселенной.

Так вот, мне интересно, сможем ли мы создать полностью автономную систему? Я думаю, нам хватит искусственной планеты (чтобы жить), искусственного спутника (для риливов) и искусственного солнца (для света и центра притяжения).
Аноним 19/01/16 Втр 05:07:34 #176 №251150 
>>251148
>Так вот, мне интересно, сможем ли мы создать полностью автономную систему?
Наверное сможем. Хотя, может, и нет.
Аноним 19/01/16 Втр 05:09:50 #177 №251151 
>>251150
Спасибо, что все для меня разъяснил.
Аноним 19/01/16 Втр 06:42:11 #178 №251160 
>>251148
А где ты собрался пилить искусственную планету, если вселенной все равно пиздец?
Аноним 19/01/16 Втр 06:43:11 #179 №251162 
>>251160
В пустоте.
Аноним 19/01/16 Втр 06:52:07 #180 №251164 
>>251160
Да ты же типичный петушок, путаешь вселенную с галактикой.
Аноним 19/01/16 Втр 07:44:58 #181 №251170 
>>251148
Сейчас непонятно откуда брать энергию когда вся она распределится по вселяшке равномерно. Когда звёзды выгорят а дыры рассосутся. В общем в сегодняшних представлениях науки пиздец неизбежен, хотя и очень нескоро.
Аноним 19/01/16 Втр 10:56:45 #182 №251220 
Может ли статься так, что есть звезды ближе, чем проксима Центавра, просто их еще не открыли из-за их неяркости?
Аноним 19/01/16 Втр 12:07:00 #183 №251262 
>>251220
>Может ли статься так, что есть звезды ближе, чем проксима Центавра, просто их еще не открыли из-за их неяркости?
Вероятность этого невелика, но да, она есть. Нахуй тебе какие-то хоббиты сплющились? Они будут крайне невелики как по яркости, так и по массе. Да та же Проксима (и Альфа) - вроде так, шваль какая-то. Поправьте, если ошибаюсь.
Аноним 19/01/16 Втр 12:17:14 #184 №251265 
>>251262
> Проксима
Карлик размером с Юпитер.
Аноним 19/01/16 Втр 12:18:55 #185 №251267 
>>251262
В смысле, очень небольшая вероятность есть. А звезды скорее всего нет.
Аноним 19/01/16 Втр 12:46:42 #186 №251271 
>>251265
Ну вот видишь, говно какое-то. В десяти световых годах приличных звезд - на полшишечки. Так что обтекай.
Аноним 19/01/16 Втр 12:48:25 #187 №251274 
Я не хочу умирать.
Аноним 19/01/16 Втр 13:03:14 #188 №251280 
>>251274
Никто не хочет. Но всем, наверное, придется, и некоторым - очень болезненно.
Аноним 19/01/16 Втр 13:26:27 #189 №251287 
>>251280
Нет. Я не хочу. И мне не придётся.
Аноним 19/01/16 Втр 16:07:04 #190 №251378 
Как та получилось что человек оказался относительно хорошо приспособлен к отсутствию гравитации? ВРоде бы любые внеземные условия укакошат человека за пару минут, а гравитация норм.
Аноним 19/01/16 Втр 16:13:25 #191 №251382 
Почему бы ПРОСТО не притащить металлический астероид в L4 Луны-Земли, раздробить его, переплавить и из этого материала изготовлять конструктивные элементы кораблей? А для топлива из пояса астероидов привезти воду.
Аноним 19/01/16 Втр 16:15:45 #192 №251384 
>>248876
Перекись водорода попробуй, отвечаю, ещё захочешь.
Аноним 19/01/16 Втр 16:15:45 #193 №251385 
>>251382
> Почему бы ПРОСТО не притащить металлический астероид в L4 Луны-Земли, раздробить его, переплавить и из этого материала изготовлять конструктивные элементы кораблей? А для топлива из пояса астероидов привезти воду.
Вези, я разрешаю.
Потому что для этого тебе придется построить в космосе немаленький и энергоемкий заводик.
Аноним 19/01/16 Втр 16:18:13 #194 №251386 
>>250974
>3 года полёта к Марсу
Ты на пердяжной тяге лететь собрался?
Аноним 19/01/16 Втр 16:19:28 #195 №251387 
>>251385
>энергоемкий
Как там Солнце, не погасло ещё?
Аноним 19/01/16 Втр 16:25:56 #196 №251391 
>>251387
> Как там Солнце, не погасло ещё?
Ну так строй, кто тебе не дает? Я вон даже свое личное благословение тебе дал. Просто в первом приближении указал трудности.
Аноним 19/01/16 Втр 16:29:35 #197 №251394 
>>251391
Ну это я так, общий план набросал, а реализовать можете сами, я даже денег не возьму.
Аноним 19/01/16 Втр 16:36:10 #198 №251399 
>>251382
в первую очередь проблема в том что придется этот астероид тащить - чем ?
Те прежде чем построить то чем тащить, надо притащить то из чего его строить.
Притаранить астероид 30м в диаметре, ну на много его не хватит, а сам по себе он уже ощутимо весит. Выискивать чтоб он сразу был весь чугуниевый (и весить еще больше), тоже надо на чемто эту разведку совершать.

В принципе L1 как место сборки, норм, годно.
С луны накидать материалов туда, и строить.
Хочу свечной заводик, в L1
Аноним 19/01/16 Втр 16:38:41 #199 №251401 
>>251399
Колбасу верни, гад!
Аноним 19/01/16 Втр 16:57:10 #200 №251422 
>>251142
>Где я могу просвящаться по астрономии преимущественно?
Купи учебник по астрономии, что ты как пидор. Алсо дохуя литературы лежит в свободном доступе в интернетах.
http://www.sai.msu.ru/neb/rw/textbook.htm
Аноним 19/01/16 Втр 19:40:36 #201 №251494 
>>251287
с хуяли?
Аноним 19/01/16 Втр 20:03:39 #202 №251504 
А нахуя вообще нужен Роскосый?
sageАноним 19/01/16 Втр 20:21:09 #203 №251507 
>>251504
>Госкорпорация "РОСКОСМОС" обеспечивает реализацию госполитики в области космической деятельности и ее нормативно-правовое регулирование, а также размещает заказы на разработку, производство и поставку космической техники и объектов космической инфраструктуры.

>В функции государственной корпорации также входит развитие международного сотрудничества в космической сфере и создание условий для использования результатов космической деятельности в социально-экономическом развитии России.


Или это был риторический вопрос?
Аноним 19/01/16 Втр 23:29:52 #204 №251596 
В войде Эридана правда очко в другую Вселенную?
Аноним 19/01/16 Втр 23:40:06 #205 №251604 
>>251596
Да, очко твой мамаши.
sageАноним 19/01/16 Втр 23:40:57 #206 №251606 
Что за быдло в треде. Пиздец. Перебанить бы вас всех, рачьё.
Аноним 20/01/16 Срд 00:02:46 #207 №251610 
Что будет, если дристануть в космосе?
sageАноним 20/01/16 Срд 00:48:00 #208 №251620 
>>251610
Из чьего-то ануса вылетишь ты.
Аноним 20/01/16 Срд 04:47:12 #209 №251666 
>>251382
>Почему бы ПРОСТО не притащить металлический астероид в L4 Луны-Земли, раздробить его
Просто не нужно ТАКОЕ количество железа на постройку: и на Земле достаточно производят. Ты ж не гигантские болванки собрался запускать. Нужны люминь, всеразличный графен, космические технологии. Высокие технологии нужны. Люди еще, что тоже ресурс.
Аноним 20/01/16 Срд 05:21:20 #210 №251669 
>>251610
Формулируй вопрос в соответствии с тематикой раздела:
>Какова дельта-V среднего человека в условиях открытого космоса при использовании различных видов топлива (горох, фасоль, ментос с кока-колой) и натощак?
Аноним 20/01/16 Срд 08:50:28 #211 №251693 
>>251669
Проиграл.
Аноним 20/01/16 Срд 08:54:24 #212 №251694 
Наверно уже спрашивали. Как решается проблема с менструацией космонавток? А то как-то не ок наверное, когда женские выделения по всему кораблю разлетаются. И второй вопрос: вот мужики дрочат в специальных гандонах, а если баба захочет в космосе - что ей делать? И неужели в космос провозят спецлитературу для дрочения?
Аноним 20/01/16 Срд 09:49:21 #213 №251701 
>>251694
Ты дурак?
Аноним 20/01/16 Срд 09:50:14 #214 №251702 
>>251694
> вот мужики дрочат в специальных гандонах, а если баба захочет в космосе - что ей делать?
дрочат только малолетние сосачерские дегенераты вроде тебя.
Аноним 20/01/16 Срд 10:29:20 #215 №251704 
>>251702
Слишком много прилагательных.
Аноним 20/01/16 Срд 10:57:51 #216 №251713 
14532766718690.jpg
>>251704
Целых два!
Аноним 20/01/16 Срд 11:05:53 #217 №251719 
>>251713
Моя есть плёхо говориль рускай. Дрочиль хорошо.
Аноним 20/01/16 Срд 18:11:38 #218 №251878 
>>251164
Увидел необоссанного долбоеба и обоссал.
Аноним 20/01/16 Срд 18:26:39 #219 №251889 
>>251148
Вселенные создаются Творцом и уничтожаются по мере завершения цикла. Всё чётко продумано, не волнуйся.
Аноним 20/01/16 Срд 18:35:59 #220 №251903 
14533041600650.jpg
игрался с новым fuji s9800 решил чекнуть небо. выдержка 8сек изо 800. диафрагма F2.9 сразу нечто примечалельное нашлось в виде штриха. фотографировал вертикально вверх. Теперь вопрос, что это за штрих? метеор или спутник?
Аноним 20/01/16 Срд 18:36:44 #221 №251904 
14533042042350.png
>>248689
> Скорее всего да, а потому вопрос: им вылают резинки, чтобы семя не разлеталось по станции? Или может резиновые вагины какие-то.
Полотенце! Они используют ёбаное полотенце!
https://www.reddit.com/comments/hbasq/do_astronauts_masturbate_in_space_xpost/
Аноним 20/01/16 Срд 18:42:06 #222 №251912 
>>251148
Бесконечно будет расширяться твой анус, а вселенная сожмётся в итоге. Сможет ли человек остановить сжатие вселенной? Вот в чём вопрос.
Аноним 20/01/16 Срд 19:55:05 #223 №251979 
>>251903
Похоже на метеор. И ты от фонаря подальше бы отошёл, вон блик какой здоровенный.
sageАноним 20/01/16 Срд 21:47:27 #224 №252116 
>>251903
Спутник, я таких по несколько штук на снимок ловлю иногда. Метеор более яркий и обычно в форме сильно-сильно вытянутого клина.
Аноним 20/01/16 Срд 21:55:26 #225 №252120 
>>251386
Память подвела, ошибся.

>>251378
Воощет это считается одной из основных проблем при том же полёте к Марсу. А ты говоришь. Кости разрушаются, сосуды ослабевают, мышцы уходят. Когда люди вывалятся на Марс они могут и не встать, даже учитывая что гравитация меньше нашей. А если встанут - ноги подломить рискуют.
Аноним 20/01/16 Срд 22:09:18 #226 №252132 
14533169586070.jpg
>>251889
>уничтожаются по мере завершения цикла
Cycle must be broken!
Аноним 20/01/16 Срд 22:31:54 #227 №252165 
Насколько сильно светит юпитер на его спутниках?
Аноним 20/01/16 Срд 22:45:21 #228 №252177 
>>251912
Распорки надо поставить железные.
Аноним 20/01/16 Срд 23:37:26 #229 №252220 
>>252165
Как OSNE BOLSHAYA YOBA
Аноним 21/01/16 Чтв 00:42:17 #230 №252311 
14533261371780.png
Аноны, укажите в чем я проебался.

Докажем, что Вселенная не бесконечна. Согласно Теории Относительности, материя не может изменяться (именно изменяться, а не двигаться по пространству) быстрее скорости света. Отсюда следует, что никакая материя не может быть бесконечной, ибо уже ограничена по своей природе. Теперь важный финт ушами - определяем что есть вселенная. Если вы считаете за вселенную само пространство, то тут никаких ограничений нет, оно не ограничено скоростью света и вполне может быть бесконечным. НО, по сути, пространство без материи ничего из себя не представляет. То есть, если даже вы будете считать вселенной именно само пространство, то у вас все равно получится ограниченное по своим размерам скопление(ия) материи в каком-то месте, а дальше будет один сплошной вакуум, что не сильно-то смахивает на бесконечность, честно говоря.
Единственным вариантом, когда бесконечная материя могла успеть появиться во вселенной - это время, когда она была в состоянии сингулярности, то есть на момент самого начала Большого Взрыва.
Аноним 21/01/16 Чтв 00:44:51 #231 №252313 
>>252311
>а дальше будет один сплошной вакуум,
Не вакуум, а просто пространство без материи конечно же.
Аноним 21/01/16 Чтв 01:00:40 #232 №252316 
14533272407750.jpg
>>252313
нихуя
пространства тоже нет
ничего нет
sageАноним 21/01/16 Чтв 03:32:28 #233 №252530 
>>252311
>укажите в чем я проебался
Отождествлять материю и пространство, подразумевая, что это одно и тоже, подменяя понятия, устраивая дешевые спекуляции, с попытками выдать бред за околонаучную теорию.
[SIDJEI] Аноним 21/01/16 Чтв 05:23:37 #234 №252534 
>>248666 (OP)
Короче я лечу со скоростью близкой к скорости света, и делаю прыжок вперед, много прыжков вперёд, да так, что достигаю скорости света, и короче я делаю ещё прыжок и лечу уже со скоростью быстрее света?
как вам открытие?
sageАноним 21/01/16 Чтв 05:28:10 #235 №252535 
>>252534
Хуйня, а не открытие.
[SIDJEI] Аноним 21/01/16 Чтв 05:38:25 #236 №252536 
>>252535
почему брат?
sageАноним 21/01/16 Чтв 05:41:38 #237 №252537 
>>252536
Ну какая-то хуета, у тебя право, поебень уровня кенгуру. Вот Специальная теория относительности - это открытие.
Аноним 21/01/16 Чтв 06:34:13 #238 №252539 
>>252530
А конретно?
[SIDJEI] Аноним 21/01/16 Чтв 06:43:23 #239 №252540 
>>252311
>>Kek
Аноним 21/01/16 Чтв 09:07:35 #240 №252562 
>>252311
Этот прав.
Аноним 21/01/16 Чтв 09:28:14 #241 №252564 
14533576945170.jpg
Что влияет на цвет полярного сияния? Какие факторы?
Аноним 21/01/16 Чтв 09:49:29 #242 №252568 
>>249523
Абсолютно чёрное тело - всего лишь модель для объяснения понятия цветовой температуры и прочих приколов. ИРЛ только ЧД ни хрена не отражает, глаз хоть и близок к АЧТ, но им не является.
Аноним 21/01/16 Чтв 09:53:56 #243 №252571 
>>252564
>Цвет полярного сияния зависит от его интенсивности. Интенсивность свечения полярного сияния определяется по принятой международной шкале в пределах I-IV баллов. Сияния с низкой интенсивностью свечения (от I до III баллов) не кажутся человеческому глазу разноцветными, так как цветовая интенсивность в них ниже порога нашего восприятия. Полярные сияния с интенсивностью в IV балла и III (у верхней границы) воспринимаются как цветные - чаще как жёлто-зелёные, реже - красные и фиолетовые. Интересно, что большая часть излучения испускается главными составляющими высоких слоёв земной атмосферы - атомарным кислородом, который окрашивает полярные сияния в желтоватые тона, придаёт им красноватую лучистость или вносит в общий спектр зелёную линию и молекулярным азотом, отвечающим за основной красный и фиолетовый цвета одного из прекраснейших небесных явлений.
Аноним 21/01/16 Чтв 10:34:19 #244 №252578 
>>252571
Спасибо.
Аноним 21/01/16 Чтв 10:59:26 #245 №252580 
Есть какие-нибудь обсерватории, где можно смотреть в телескоп онлайн в прямом эфире?
Аноним 21/01/16 Чтв 11:08:50 #246 №252581 
Существуют ли зеленые звезды? И если нет, то почему мы их не видим?
Аноним 21/01/16 Чтв 11:25:39 #247 №252585 
14533647393020.gif
>>252581
Ты не захочешь увидеть зелёную звезду, уверяю.
Аноним 21/01/16 Чтв 12:04:22 #248 №252592 
14533670623320.jpg
>>252581
внезапно солнце по спектральному классу относится к зеленым
Аноним 21/01/16 Чтв 12:39:10 #249 №252607 
>>249175
А через что же я Titan's Fall покупал?
Аноним 21/01/16 Чтв 12:42:52 #250 №252608 
>>252607
Оригин есть, подписки (или чего там рекламируют) нет. По крайней мере в РБ.
Аноним 21/01/16 Чтв 13:08:49 #251 №252615 
>>252311
Ты не подставил понятие время. Если не существует времени, то и материи нет.
Материя- волны на трехмерной простыне. Простыня бесконечна, она- основа и база для всего, материя- колебания на этой простыне, или скорее даже разность потенциалов.
Аноним 21/01/16 Чтв 13:24:34 #252 №252618 
>>252615
Как твое описание пространства-времени противоречит посту?
Аноним 21/01/16 Чтв 13:43:51 #253 №252620 
>>248666 (OP)
Че там с девятой планетой?
Аноним 21/01/16 Чтв 13:47:31 #254 №252622 
>>252620
>>251993
Аноним 21/01/16 Чтв 14:14:33 #255 №252630 
>>252618
Он же говорил про бесконечное пространство.
А пространство- просто разность энергии на какой-то основе.
Аноним 21/01/16 Чтв 15:05:00 #256 №252648 
>>252630
Как я понял, там прастранство не так важно. Главное, что следуя его выводом можно "доказать" ограниченость материи во вселенной.
Аноним 21/01/16 Чтв 20:09:08 #257 №252744 
Почему наша солнечная система да и галактика в целом вращаются в одной плоскости? Читал про момент сохранения импульса, не очень понял как это связано.
Аноним 21/01/16 Чтв 22:02:53 #258 №252783 
>>252744
>Читал про момент сохранения импульса, не очень понял как это связано.
Напрямую: изначальное движение было и оно никуда не могло исчезнуть.
Аноним 21/01/16 Чтв 22:50:18 #259 №252805 
>>250355
Имел ввиду, что во время излучения создается частица-античастица, как бы если частицу гравитация выпустила, то античастица должна с ней слиться в объятьях братских и к хуям аннигилировать. Либо частица попадает в грав. ловушку и у нас остается античастица соло, мы не можем регистрировать наличие излучения античастиц - регистратор порвется, только косвено - распидорашивание чего-то ничем, рядом с чд.
Или я не так понимаю?
Аноним 21/01/16 Чтв 23:23:01 #260 №252831 
>>252744
Наклонение Солнечной системы к плоскости галактики 60 градусов.
Аноним 22/01/16 Птн 00:14:23 #261 №252860 
>>252805
Нет, не так. Вообще излучение Хокинга имеет довольно сложное объяснение.
Вот объяснение тебе, удачи
https://youtu.be/ri3ElJwZ-Jo?t=23m43s
Аноним 22/01/16 Птн 00:56:25 #262 №252866 
>>252860
Излучение Хокинга это одна из гипотетических моделей, просто потому что сам Хокинг типа крутой авторитет, эту модель и пиарят больше всего, не обижать же инвалида, в самом-то деле.
Аноним 22/01/16 Птн 01:11:46 #263 №252868 
>>252866
>потому что сам Хокинг типа крутой авторитет, эту модель и пиарят больше всего
Хокинг в своей "Краткой истории времени" прямым текстом написал, что эту гипотезу он честно спиздил у Зельдовича и Старобинского, когда был в Москве в 1973-м году. Единственное, что ему не понравилось - это кривоватый математический аппарат, который те использовали.
Аноним 22/01/16 Птн 01:13:59 #264 №252869 
>>249509
Англичанка на халяву слетала
Аноним 22/01/16 Птн 02:59:25 #265 №252941 
Слоупок на связи.
Посмотрел этот ваш Марсианин. Такая-то беспомощная пропоганда, ну да хуй с нею. Вопрос вот в чем. Возможна ли в принципе такая конфигурация планет чтобы двумя гравиманеврами можно было слетать от Земли до Марса и обратно без двигателей?
Аноним 22/01/16 Птн 03:08:09 #266 №252947 
>>252941
>без двигателей
Без двигателей - нет. Впрочем, в Марсианине двигатели использовались.
Аноним 22/01/16 Птн 03:15:12 #267 №252951 
>>252947
Окей, для принципиальных скажу так. Без существенной коррекции траектории на которую тупо не хватит топлива.
Аноним 22/01/16 Птн 03:44:05 #268 №252960 
>>252951
Аффтар говорит, что все орбиты посчитал. Дату говорить отказывается.
Аноним 22/01/16 Птн 03:59:40 #269 №252965 
https://en.wikipedia.org/wiki/Free_return_trajectory
https://en.wikipedia.org/wiki/Mars_cycler
Аноним 22/01/16 Птн 04:12:25 #270 №252969 
>>248666 (OP)
https://www.youtube.com/watch?v=kydGM0GiPhs
22:46 - 23:55

Если представить, что вся материя прозрачна, значит ли это, что в какую бы точку Вселенной не посмотрел бы наблюдатель - его взгляд "устремится" в собственный затылок?
Аноним 22/01/16 Птн 04:21:42 #271 №252972 
>>252969
Нет.
Аноним 22/01/16 Птн 04:42:50 #272 №252976 
14534269702010.jpg
Спейсач, была ли новая годнота после сериала Space time odyssey с моим любимым ниггером из оп-поста? Одиссея просто охуительно хороша и йоба и ЖЗЛ и никакого пиздежа ради пафоса, максимально просто суть научная. А после еще ничего не выходило?
Аноним 22/01/16 Птн 05:19:10 #273 №252978 
Если выстрелить из пушки на Луне в сторону Земли, то снаряд долетит до Земли или станет спутником?
Аноним 22/01/16 Птн 05:25:59 #274 №252979 
>>252978
Зависит от начальной скорости
Аноним 22/01/16 Птн 05:58:44 #275 №252981 
>>252976
Нет. Тайсон сейчас хуярит подкаст "StarTalk Radio".
Но если не можешь в инглиш, то в пролете.
https://soundcloud.com/startalk
Аноним 22/01/16 Птн 06:42:34 #276 №252984 
14534341544910.png
>>252972
И всё же, процитирую: "В случае со Вселенной, астрономы считают, что она может быть закруглённой, как баскетбольный мяч, но она настолько огромна, что мы можем видеть лишь небольшую часть; поэтому то что мы видим, выглядит плоским - несмотря на присущую ей изогнутость."
Аноним 22/01/16 Птн 06:44:28 #277 №252985 
>>252116
>>251979
>>251903
это вспышка иридиума (или другого спутника).
Если ОП скажет, где был север, могу это подтвердить.
Аноним 22/01/16 Птн 06:54:09 #278 №252986 
>>252984
Если что-то цитируешь, указывай источник. Если вселенная закругленная, то она обладает постоянной положительной кривизной, а на это ничего не указывает, более того, в последнее время собрано довольно много данных, указывающих на то, что Вселенная скорее всего плоская.
Аноним 22/01/16 Птн 06:57:18 #279 №252987 
>>252985
Что ты будешь подтверждать, болезный, что бы сказать, что это вспышка Иридиума нужно знать время съемки до секунды и место.
То, что это спутник и так понятно. И похую где север, у спутника может быть любое наклонение орбиты.
Аноним 22/01/16 Птн 12:14:28 #280 №253060 
1) Если у Союза не сработают тормозные движки, космонавты выживут? Сломают кости?
2)Когда перестали катапультироваться из СА?
Аноним 22/01/16 Птн 12:34:13 #281 №253069 
3) Почему первые полеты оканчивались в Поволжье, а дальнейшие в Казахстане?
Аноним 22/01/16 Птн 12:44:31 #282 №253073 
4)На каком этапе СА отделяется от других отсеков
5)Двигатели СА используются только у земли, или как-то маневрируют/гасят скорость на орбите или в процессе входа в атмосферу?
6)Топливные баки у САС свои, или используются какие-то другие - от СА или ПАО?
Аноним 22/01/16 Птн 12:47:39 #283 №253076 
Какая самая тяжолая ракетаноситель?
Аноним 22/01/16 Птн 13:10:14 #284 №253081 
>>252981
Могу, спасибо няша. Жаль что только подкаст, очень уж доставило визуальное отображение в одиссейке
Аноним 22/01/16 Птн 13:13:16 #285 №253084 
>>253060
> 1) Если у Союза не сработают тормозные движки, космонавты выживут? Сломают кости?
Выживут, потом сломают кости тем, кто накосячил с движками. Вроде как ложементы должны обеспечивать здоровье даже без ДМП.
> 2)Когда перестали катапультироваться из СА?
После "Востока". На "Восходе" вообще никакой САС не было.
>>253073
> 4)На каком этапе СА отделяется от других отсеков
Перед входом в плотные слои.
> 5)Двигатели СА используются только у земли, или как-то маневрируют/гасят скорость на орбите или в процессе входа в атмосферу?
На орбите используются двигатели приборно-агрегатного отсека, при посадке используются твердотопливные движки, стоящие на самом СА.
> 6)Топливные баки у САС свои, или используются какие-то другие - от СА или ПАО?
Там нет топливных баков, все САС твердотопливные, только Маск сейчас выебывается.
Аноним 22/01/16 Птн 13:25:50 #286 №253088 
>>253084
>только Маск сейчас выебывается
он не выебывается, у него выбор не большой, ему надо совместить чтобы все в одном было, вот он и делает, как САС я хз(надежность и вероятность сгореть в гептиле), очко жим жим, а так норм если будет норм.
Аноним 22/01/16 Птн 13:26:24 #287 №253089 
14534583847750.jpg
>>253076
Твая момаша)))
Аноним 22/01/16 Птн 15:50:10 #288 №253129 
>>252986
>скорее всего плоская.
Что под плоскостью и над плоскостью?
Аноним 22/01/16 Птн 17:05:25 #289 №253149 
Предположим, варп-двигатель возможен. Допустим также, к тебе в руки попался корабль с работающим варп-двигателем.
Поздравляю! Теперь ты можешь фактически передвигаться со скоростью 655 200 000 километров в секунду и добраться за полдня до самой близкой звёздной системы - альфа Центавра - достичь центра галактики Млечный путь за десять лет и так далее.
В какое место нашей огромной Вселенной ты отправишься первым делом, мой друг?
Аноним 22/01/16 Птн 17:16:07 #290 №253156 
>>253149
Отправлю зонд к центру Галактики чтобы проверить существование чёрных дыр
Аноним 22/01/16 Птн 17:17:47 #291 №253160 
Во время лунной гонки САСы ставили даже на протоны и спасали грузы. Почему сейчас их туда и на другие ракеты не ставят?
Аноним 22/01/16 Птн 17:21:32 #292 №253163 
>>253160
Уменьшает ПН.
Аноним 22/01/16 Птн 17:21:48 #293 №253164 
>>253129
Не знаю.
Аноним 22/01/16 Птн 17:34:21 #294 №253171 
>>253156
А сам почему не полетишь? И не лучше ли отправится к потенциально пригодным для жизни мирам, Kepler 62-e?
Аноним 22/01/16 Птн 18:08:47 #295 №253179 
>>253129
>Что под плоскостью и над плоскостью?
Абсолютно ничего: там нет пространства. Кроме того, пространство может быть любой хитровыебаной формы, должно быть лишь гомеоморфно плоскости.
Аноним 22/01/16 Птн 18:11:49 #296 №253182 
>>253179
Ты лох.
Аноним 22/01/16 Птн 19:00:53 #297 №253207 
>>253163
Ну и что, зато миллиардный груз спасается.
Аноним 22/01/16 Птн 19:10:22 #298 №253211 
>>252986
>Если что-то цитируешь, указывай источник.

А источник вот >>252969 - видео. Таймкод указан. И пикча оттуда. Такие дела.
Аноним 22/01/16 Птн 19:19:31 #299 №253216 
>>253182
Петуха забыли спросить.
Аноним 22/01/16 Птн 19:22:08 #300 №253218 
14534797285910.jpg
>>253179
Сфера без точки гомеоморфна плоскости.
https://otvet.mail.ru/question/32598770
Аноним 22/01/16 Птн 19:28:18 #301 №253224 
Или не? На пикче - стереографическая проекция.

Алсо, Клиффорд, Хопф, и многабукафф:
http://www.liveinternet.ru/users/5146949/post329517496/

>>253218-кун
Аноним 22/01/16 Птн 19:37:18 #302 №253226 
>>253129
Она топологически плоская. Это не значит, что Вселенная похожа на двумерный блин, поскольку она трехмерная, а не двумерная. Поэтому вопрос "что сверху и снизу" абсолютно не корректен.
Аноним 22/01/16 Птн 19:45:50 #303 №253229 
Почему идеальная форма во Вселенной - шар?
Аноним 22/01/16 Птн 19:51:51 #304 №253231 
>>253229
>Почему идеальная форма во Вселенной - шар?
Что значит "идеальная"? Если ты про планеты и звезды - то потому, что поле силы гравитации - сферически симметричное.
Аноним 22/01/16 Птн 19:55:31 #305 №253232 
>>253231
Начиная от частиц в веществах и заканчивая мегаомегачёрными дырами - везде форма шара.

>сферически симметричное
Что?
Аноним 22/01/16 Птн 19:55:50 #306 №253233 
>>253226
Однако Хопф занимался более общей задачей – построением отображения трехмерной сферической поверхности или 3-сферы, находящейся в 4-мерном пространстве, на более привычное нам 3-мерное евклидово пространство, которое принято именовать плоским и обозначать R3.

Хопф для этой цели применил известную в географии и геометрии стереографическую проекцию, которая при отображении сохраняет углы между прямыми (это называется конформное преобразование), а окружности переводит также в окружности или прямые (иначе, окружности бесконечного радиуса).

Паста отсюда: https://otvet.mail.ru/question/32598770

Таким образом, суть >>252969-вопроса можно переформулировать так:
Является ли прямая, проходящая от наблюдателя к любой точке пространства (исключая самого наблюдателя, т. к. сфера без точки гомеоморфна плоскости >>253218) - окружностью, бесконечного радиуса?
Аноним 22/01/16 Птн 19:57:46 #307 №253234 
>>253232
>Начиная от частиц в веществах и заканчивая мегаомегачёрными дырами - везде форма шара.
Атом - бильярдный шар?
Аноним 22/01/16 Птн 20:00:06 #308 №253235 
14534820070060.png
>>253234
Достаточно близок к этому.
Аноним 22/01/16 Птн 20:01:20 #309 №253236 
>>253235
>21-й век.
>Планетарная модель атома.
Пишы исчо.
Аноним 22/01/16 Птн 20:03:27 #310 №253237 
>>253236
Я просил пояснить, а не отвечать в духе "твоя теория - не теория". Ты уже отписал про сферическую симметрию. Я покурил гугл, но мало что понял даже на английском. Ты пояснять будешь?
Аноним 22/01/16 Птн 20:07:45 #311 №253239 
>>253236
А впрочем, иди нахуй. Гугл дал ответ.
Аноним 22/01/16 Птн 20:10:00 #312 №253241 
>>253237
>Ты пояснять будешь?
Ну ок. Представь, что в произвольной точке пространства сосредоточена масса, гравитация от этого объекта будет распространяется во все стороны с одинаковой силой, поэтому что бы не собиралось вокруг точки оно будет принимать сферическую, или околосферическую форму.

Модель частиц в виде шариков - это абстрактное упрощение, имеющее с квантовой реальностью мало общего.
Аноним 22/01/16 Птн 20:12:34 #313 №253244 
>>253233
Теории гомотопических групп в спейсаче, ну охуеть, после всех этих вопросов "как срать в космосе"
>Является ли прямая, проходящая от наблюдателя к любой точке пространства (исключая самого наблюдателя, т. к. сфера без точки гомеоморфна плоскости >>253218) - окружностью, бесконечного радиуса?
Да является, но кто сказал, что Вселенная имеет форму сферы?
Аноним 22/01/16 Птн 20:24:42 #314 №253247 
Почему государства так мало вкладываются в космос? Почему бы рашке весь свой бюджет не пустить на освоение Луны, построить купола с искусственной средой и ебаться там с церквями, добывать энергию из гелия-3 и не бояться ничего, никакой мировой общественности?
Аноним 22/01/16 Птн 20:39:09 #315 №253253 
14534843491630.jpg
>>253247
>Почему бы рашке весь свой бюджет не пустить на освоение Луны
Потому что нужно жрат, строить, где-то работать и вообще жить в целом. Если бы люди были бы более спокойные и мирные, можно было бы не тратить деньги на оборонку и тратить их на космос и фундаментальные исследования, но увы. Имеем, что имеем.
Аноним 22/01/16 Птн 20:42:33 #316 №253255 
>>253244
>кто сказал, что Вселенная имеет форму сферы?
>>252984
>>253211
>>253218
Но в основном - дядька на видосе по тайм-коду >>252969 - поста

>Да является
Итак, Ваш окончательный ответ на >>252969 ?
Аноним 22/01/16 Птн 20:51:31 #317 №253258 
14534850919060.jpg
>>253247
Потому что, радивация. Магнитное поле Земли защищает.
Выше 24000 км над Землёй радиация убивает всё живое.
Пояса Ван Аллена. Флаг на Луне развивается от солнечного ветра.

Аноним 22/01/16 Птн 20:54:58 #318 №253260 
>>253255
Да хули вы спорите? Сейчас наиболее принята теория, что вселенная - топологически плоская. Саму топологию можно в другом месте пообсуждать.
Аноним 22/01/16 Птн 20:57:39 #319 №253264 
14534854596020.jpg
>>253247
Лучше, японский Luna Ring, чем гелий-3.

Cодержание гелия-3 в реголите составляет ~1 г на 100 т.
Аноним 22/01/16 Птн 21:01:18 #320 №253267 
>>253260
>топологически плоская
Куб? Сфера? Затылок увижу?
Аноним 22/01/16 Птн 21:08:09 #321 №253270 
>>253264
Ну так вперёд! Будешь со мной добывать?
Аноним 22/01/16 Птн 21:10:21 #322 №253271 
>>253260
>наиболее принята теория
Какая такая теория?
>топологически плоская
Гипотеза Пуанкаре
>вселенная
Вселенная
Аноним 22/01/16 Птн 21:11:05 #323 №253273 
>>253270
Там радивация. Голактеко в опасносте.
Аноним 22/01/16 Птн 21:11:06 #324 №253274 
>>253255
Блядь, тебе говорят, что Вселенная топологически плоская, а ее форма не определена. Что бы там тебе не писали на мейл ру, в твоем кине тот же Филипенко говорит, что она плоская, но может быть нет, хотя сейчас склоняются, что блядь, да, плоская. И вот почему:
https://youtu.be/G9ttCX29BZI?t=33m19s

И даже говоря о том, что она может быть изогнута, не имеется ввиду ее конкретную форму, сферы там, или куба. Ее форма не определена.
Аноним 22/01/16 Птн 21:23:49 #325 №253287 
>>253271
>Гипотеза Пуанкаре
Она-то здесь причем? И научись ставить точки в конце предложения перед тем, как выебываться правописанием.
Аноним 22/01/16 Птн 21:32:04 #326 №253291 
>>253274
>топологически плоская
>форма не определена
Гипотеза Пуанкаре.

>в твоем кине тот же Филипенко говорит, что она плоская
Он говорит, дословно: "части, которые мы видим - эти части являются вполне плоскими."

"То, что мы видим - выглядит плоским, несмотря на пресущую ей изогнутость."
Аноним 22/01/16 Птн 21:38:50 #327 №253294 
>>253291
Присущую, мудак. И точки перед кавычками в твоем случае не нужны.
Аноним 22/01/16 Птн 21:44:39 #328 №253301 
>>253287
>Она-то здесь причем?
При том, что она для всякого односвязного компактного трёхмерного многообразия без края.
>И научись ставить точки в конце предложения
То не конец предложения, а фикс.
>выебываться
>срать в космосе
>научись
>в другом месте
>хули
>вы
>Блядь
Аноним 22/01/16 Птн 21:46:22 #329 №253302 
>>253301
>При том, что она для всякого односвязного компактного трёхмерного многообразия без края.
И что?
Аноним 22/01/16 Птн 21:50:47 #330 №253305 
>>253294
>точки перед кавычками
>не нужны
Попизди мне тут.
http://pogugli.com/?112480
>мудак

>Присущую
Ты о чём ваще, сынок?
Аноним 22/01/16 Птн 21:51:09 #331 №253306 
>>253302
Вот и то.
Аноним 22/01/16 Птн 22:08:04 #332 №253317 
>>253273
Но любви не страшна радивация! Ты согласен?
Аноним 22/01/16 Птн 22:11:00 #333 №253319 
14534898603150.jpg
>>253317
Страшна. Она вызывает мутации, ионизируя атомы в азотистых основаниях днк хромосом в ядрах клеток.
Аноним 22/01/16 Птн 22:13:33 #334 №253324 
>>253306
Быстро слился.
Аноним 22/01/16 Птн 22:15:48 #335 №253325 
>>253319
как хуево основания нарисованы
Аноним 22/01/16 Птн 22:18:07 #336 №253327 
>>253324
Чё? Неси пруф края.
Аноним 22/01/16 Птн 22:20:29 #337 №253331 
>>253319
То есть я мутантом стану?
Аноним 22/01/16 Птн 22:22:14 #338 №253332 
>>253331
Не, просто выскочит какая-нибудь пиздецома.
Аноним 22/01/16 Птн 22:26:01 #339 №253338 
>>253327
Да нет, это ты неси док-ва того, что пространство Вселенной:
- односвязно
- компактно
- трехмерно
- без края
Аноним 22/01/16 Птн 22:29:37 #340 №253342 
>>253338
Какие доказательства?
Аноним 22/01/16 Птн 22:38:20 #341 №253346 
>>253338
1. См. определения.
2. Топологически плоскими являются наблюдаемые участки пространства, а не Вселенная вцелом.
Аноним 22/01/16 Птн 22:40:46 #342 №253349 
>>253332
Так классно же!
Аноним 22/01/16 Птн 22:48:13 #343 №253353 
>>253291
>эти части являются вполне плоскими
Краусса я тебе нахуя скинул? Там доступно, для дебилов объяснено, что вся Вселенная плоская, а не кусочки, которые мы видим.
>Гипотеза Пуанкаре.
Повествует только об односвязных пространствах. Если Вселенная - сфера, то окей, а если ебучий тор, то ты и Пуанкере идете нахуй со всеми гипотезами, блядь, я заебался пиздеть одно и тоже, хули ты себе в башку вбил, что Вселенная - сфера? Тебе боженька нашептал что ли?
Аноним 22/01/16 Птн 22:48:33 #344 №253354 
Хочу стать мутантом. Кто со мной?
Аноним 22/01/16 Птн 22:54:36 #345 №253358 
14534924764120.jpg
14534924764131.jpg
14534924764152.jpg
14534924764153.jpg
>>253349
Классно?
Аноним 22/01/16 Птн 23:00:33 #346 №253364 
>>253358
Я вообще хотел стать бессмертным, но это тоже сойдёт!
Аноним 22/01/16 Птн 23:03:59 #347 №253366 
>>253346
Ты уж определись, плоское ли пространство, или смотреть какие-то там определения?
Аноним 22/01/16 Птн 23:15:14 #348 №253372 
14534937141670.jpg
>>253353
Люди ржут с треугольничков Краусса?

>хули ты себе в башку вбил, что Вселенная - сфера? Тебе боженька нашептал что ли? сингулярность -> БВ -> инфляция -> сфера.
Это логично.

У меня сейчас такая картинка в башке странная - куда не глянь, увидишь затылок. (если материя считать прозрачной).
Мол любая прямая - это окружность бесконечного радиуса. А центр окружности - везде и нигде одновременно. Лол.
Может быть, я представил 4мерную сферу?
Аноним 22/01/16 Птн 23:17:05 #349 №253374 
>>253366
Изнутри, оно конечно, видится (эксперементально) плоским.
Аноним 22/01/16 Птн 23:24:41 #350 №253377 
>>253372
>БВ -> инфляция -> сфера.
>Это логично.
В этом нет никакой логики. Мало того, что ты, как пятиклассник, представляешь себе БВ в виде взрыва гранаты в пустой комнате, так еще и непонятно, с чего бы это пространство должно расширяться сферически симметрично?
Аноним 22/01/16 Птн 23:31:45 #351 №253383 
>>253377
>с чего бы это пространство должно расширяться сферически симметрично

1с - во все стороны. Расширение границы видимой Вселенной с момента БВ. Как-то так.
Аноним 22/01/16 Птн 23:34:28 #352 №253385 
Ясно, очередной дебил.
Аноним 23/01/16 Суб 00:27:10 #353 №253406 
Прикроет ли нас магнитное поле Солнце, случись нашей системке попасть под гамма-вспышку? Почему-то при разговорах о таком сценарии околонаучные фрики не учитывают, что у Солнышка тоже есть своё магнитное поле, и немаленькое, всё больше говорят исключительно о земном.
Аноним 23/01/16 Суб 00:44:16 #354 №253408 
>>253406
>гамма-вспышку
Сколько джоулей предполагаешь получить?
Аноним 23/01/16 Суб 00:55:02 #355 №253410 
>>253408
Эээ, ну, я не знаю, обычная такая гамма-вспышка... Средняя такая.
Аноним 23/01/16 Суб 00:56:20 #356 №253411 
>>253406
>немаленькое
Заткнись и считай!

магнитное поле Солнца, имеет среднюю напряжённость на уровне фотосферы порядка нескольких гаусс.

Напряженность магнитного поля Земли - 0.7 Гаусс в полярных районах.
Аноним 23/01/16 Суб 01:01:22 #357 №253412 
>>253411
Ну что за хуйня. Какая дурацкая материнская звезда нам досталась.
Аноним 23/01/16 Суб 01:04:47 #358 №253413 
>>253410
>Эээ, ну, я не знаю, обычная такая гамма-вспышка... Средняя такая.
Регистрируются примерно раз в день. Куда уж чаще?
Аноним 23/01/16 Суб 02:11:20 #359 №253433 
14535042807860.gif
>>253406
>Экранирование гамма-излучения магнитным полем.

Охуеть, у нас тут новое слово в физике: отклонение гамма-излучения магнитными полями.
Аноним 23/01/16 Суб 03:03:28 #360 №253471 
>>253383
>границы видимой Вселенной
Это ты приплел, не понимая сути. Видимая Вселенная - это небольшой кусочек, который мы видим, всего-то. Из этого не следует ровном счетом нихуя.
>1с - во все стороны
А как должно быть при одной и той же постоянной Хаббла, во всей Вселенной?

>>253372
>БВ -> инфляция -> сфера.
>Это логично.

Это логично в твоей голове, которая как раз потеряла инфляцию.
Аноним 23/01/16 Суб 03:08:30 #361 №253476 
Найдена вроде как самая большая звезда вселенной https://en.wikipedia.org/wiki/ASASSN-15lh , но там размер не указан, во сколько раз она больше второй по размеру VY Большого пса?
Аноним 23/01/16 Суб 03:55:31 #362 №253517 
>>253476
>вроде как самая большая звезда вселенной
Самая яркая сверхновая. Что такое сверхновая можешь почитать в википедии.
Более того, звезда вспыхнула в 3800000 световых годах, в другой галактике. На таком расстоянии можно узнать только стандартные свечи. А эта хуйня бахнула в 1000 раз ярче, чем типичная сверхновая, так что о стандартных свечах говорить не приходится. Не говоря о том, что измерять там особо нечего. Даже самая толстая нейтронная звезда редко имеет радиус больше 20 км.
Аноним 23/01/16 Суб 09:50:59 #363 №253565 
Почему не используется такое перспективное топливо как пентаборан?
Аноним 23/01/16 Суб 10:29:55 #364 №253566 
>>253565
>Проблемы с данным топливом заключались в его токсичности, в его свойстве вспыхивать при контакте с воздухом и токсичности продуктов горения. Безопасный и дешёвый способ утилизации пентаборана был изобретён только в 2000 году — он основан на гидролизе пентаборана водяным паром, в результате которого получаются газообразный водород и раствор борной кислоты (Н3BО3). Это позволило избавиться от накопленных запасов пентаборана в США.
Аноним 23/01/16 Суб 10:34:29 #365 №253567 
>>253566
Гептил тоже опасен, но используют
Аноним 23/01/16 Суб 10:40:39 #366 №253569 
>>253149
Полечу в предполагаемое местонахождение СУПЕРЗЕМЛИ в нашей Солнечной системе.
Аноним 23/01/16 Суб 11:00:17 #367 №253571 
>>253149
На Титан
Аноним 23/01/16 Суб 11:11:27 #368 №253572 
С МКС выше шанс наблюдать солнечное и лунное затмение, или ниже? Это вообще возможно?
Аноним 23/01/16 Суб 13:48:49 #369 №253593 
>>253471
>Видимая Вселенная - это небольшой кусочек, который мы видим, всего-то.
В которой Хаббл видит галактики, возрастом ~13 млрд. лет.

>1с - во все стороны
>А как должно быть при одной и той же постоянной Хаббла, во всей Вселенной?
Ну... Так ли это?


>Это логично в твоей голове, которая как раз потеряла инфляцию.
Пространство: БВ -> 1с -> сфера.
Материя: БВ -> менее 1c -> контент внутри сферы.

Т. е. тормозит расширение материи - гравитация.
Инфляция же, обратный во времени процесс,
но видимый сейчас для наблюдателя, наблюдающего ещё те события, на тех высоких скоростях,
и т.к. свет них дошёл лишь сейчас, те скорости расширения - не соответствуют сегодняшним, меньшим.
Вот те и инфляция.

Всё верно?
Аноним 23/01/16 Суб 14:06:41 #370 №253597 
>>253433
ну раз ты такой умный можешь рассматривать это как рассеяние гамма излучения на электронах находящихся в магнитной ловушке.
Аноним 23/01/16 Суб 15:58:02 #371 №253613 
>>252132
Лол.
sageАноним 23/01/16 Суб 19:56:06 #372 №253659 
>>253593
>В которой Хаббл видит галактики, возрастом ~13 млрд. лет.
Доооо, и конечно же Земля находится в самом центре Вселенной. Пуп Мира так сказать.
>Ну... Так ли это?
А что, не так?

Аноним 23/01/16 Суб 21:44:29 #373 №253680 
Привет космоанону!
У меня не вопрос, а скорее реквест.
Тучу лет назад, бродя по википедии наткнулся на статью про какого-то немецкого учёного, который типа разработал теорию о том, что если создать магнитное поле достаточной интенсивности и сильно его раскрутить, то можно открыть портал в параллельное измерение. Это конечно НФ хуйня, типа пузыря альбукерке, но щас смотрю лекции Попова, где он про нейтронные звёзды рассказывает, про их магнитные поля в миллиарды гауссов ну и короче чёт вспомнился тот немец.
Может кто его фамилию доставить? С меня чё-нибудь.
Аноним 23/01/16 Суб 21:51:24 #374 №253681 
>>253680
Георг Гартман? Георг Хартман? Максом Планк?
Быть может, Карл Шварцшильд?
Аноним 23/01/16 Суб 21:53:47 #375 №253682 
>>253659
>Доооо, и конечно же Земля находится в самом центре Вселенной.
В самом центре, наблюдаемой с Земли, Вселенной.
>Пуп Мира так сказать.
Не?

>А что, не так?
Так-то, оно вроде-бы так. Вроде.
Аноним 23/01/16 Суб 21:55:32 #376 №253683 
14535753323370.jpg
>>253681
>Георг Хартман?
>Макс Планк?
>Карл Шварцшильд?
fix
Аноним 23/01/16 Суб 22:41:03 #377 №253694 
>>253681
нет, не они. Но помню, что он помер где-то в 80-х и у него вся теория очень сильно перекликается с теорией струн и её многомерностью.
sageАноним 23/01/16 Суб 23:07:44 #378 №253699 
>>253682
>Не?
С хуя ли?
>В самом центре, наблюдаемой с Земли, Вселенной.
Наблюдаемая вселенная и Вселенная - две большие разницы.
>Вроде.
Понятно, теперь у нас не только Земля - пуп Вселенной и центр Мироздания, так еще и Вселенная анизотропна.
Аноним 24/01/16 Вск 00:04:46 #379 №253719 
14535830863030.jpg
>>253699
>С хуя ли?
х3
>Наблюдаемая вселенная и Вселенная - две большие разницы.
Даже если БВ произошёл внутри объемлющей Вселенной,
возраст галактики UDFj-39546284, существовавшей 13,4 миллиарда лет назад, снимок которой был сделан космическим телескопом «Хаббл» - не так уж и сильно отличается от возраста наблюдаемой вселенной с момента БВ.
Речь же идёт о границе расширяющегося пространства, порождённого при БВ, скорости его расширения, и форме оного.

Предполагается, что скорость расширения равна 1с, и как следствие, форма пространства - сферическая, но видимая изнутри - плоской.

>Понятно, теперь у нас не только Земля - пуп Вселенной и центр Мироздания так еще и...
Отчасти, да.
Просто оставлю это здесь:
https://www.youtube.com/watch?v=H3oKzqc1z9E

>... Вселенная анизотропна.
Анизотропна в отношении чего?
Аноним 24/01/16 Вск 00:11:49 #380 №253729 
>>253719
>Просто оставлю это здесь:
И вот это, тоже, вброшу.
https://www.youtube.com/watch?v=waNgpOz295Y
Аноним 24/01/16 Вск 00:38:33 #381 №253739 
Что происходило с большим оранжевым баком шаттла?
Аноним 24/01/16 Вск 01:01:28 #382 №253744 
>>253739
Сгорал в атмосфере.
Аноним 24/01/16 Вск 02:00:16 #383 №253759 
>>253719
>Даже если БВ произошёл внутри объемлющей Вселенной
>Земля, как центр БВ, который произошел локально, уже в некоей существующей Вселенной
>скорость расширения равна 1с

Пиздец у тебя каша в голове.
sageАноним 24/01/16 Вск 02:17:36 #384 №253764 
>>253597
Вероятность комптоновского рассеяния пропорционально количеству электронов в объеме. Найдешь столько электронов, что бы рассеять гамма всплеск, учитывая его энергетику, в космическом пространстве?
Аноним 24/01/16 Вск 02:43:02 #385 №253770 
>>253759
Да, такое бывает. И чё терь?
Аноним 24/01/16 Вск 02:44:42 #386 №253772 
>>253770
Ну если тебя это устраивает - ничего. Если ты хочешь это поправить, читай, систематизируй знания, приводи в голову в порядок.
Аноним 24/01/16 Вск 02:49:12 #387 №253777 
>>253759
>Земля, как центр БВ, который произошел локально, уже в некоей существующей Вселенной
Чего? Хоть ты и хуйню написал, но тебе сюда - >>253729
Аноним 24/01/16 Вск 02:54:22 #388 №253780 DELETED
>>253777
Почитай вот эти охуительные истории вот этого полуебка >>253719

А от меня отъебись.
Аноним 24/01/16 Вск 02:56:27 #389 №253781 
>>253764
отлична от нуля, как я и предсказывал - нобелевку мне, быссроо.
>рассеяния пропорционально количеству электронов в объеме
и да я чето поскромничал, надо было сразу ионов сказать, те плазмы
>Найдешь столько электронов, что бы рассеять гамма всплеск
да, фотосфера солнца, там дохуища электронов. Фотосфера практически не прозрачна для гамма.

Если копнуть поглубже - там дохуища дохуищь электронов.
И естественно замый важный момент - из возможных в нашей системе, самый пиздатый из пиздатых экранов - да, это непосредственно само солнце.
Никакая гамма-вспышка его не просветит. Подчеркну - _никакая_ гамма.

>учитывая его энергетику
сколько в граммах

но ты реально хочешь пободаться вокруг этого изначально тупого вопроса ? >>253406
>Почему-то при разговорах о таком сценарии околонаучные фрики не учитывают, что у Солнышка тоже есть своё магнитное поле
>Почему-то при разговорах о таком сценарии околонаучные фрики не учитывают
когда вопрос собственно выше, и он сразу должен был получить ответ - потомучто фрики и тупые и умные.
умные потому, что знают, что человеков за пределами земной орбиты нету. Нету человеков, нету и обсуждений.

>Прикроет ли нас магнитное поле Солнце
нет, никаким макаром, так как там где человеки есть, магнитное поле земли, больше чем магнитное поле солнца.

также следовало просто рассеять заблуждение или ошибку формулировки анона
>магнитное vs гамма излучение?
гамма излучение с магнитным полем как бы совокупляются, и никакого урона друг другу не наносят - так как это одно и тоже, просто у простого магнитного поля, о котором ты говоришь - частотат близка к нулю.
А что же подразумевают, когда говорят о вспышках, обычно говорят о солнечных вспышках, так это то, что наше солнышко выплевывает из себя потоки плазмы и нейтронов. И нейтроны не взаимодействуют с магнитнм полем, в силу отсутвия заряда, но период их полураспада 15 минут (зависит от энергии), и они могут распадаться внутри магнитного поля(на протон и электрон), образуя частицы которые уже могут взаимодействовать с полем. А плазма изначально может взаимодействовать с магнитным полем, и взаимодействует - что проявляется к примеру в северных сияниях. Это один из маоентов, о котором и говорят, когда говорят о вспышках. О чем вы можете почитать более подробно в википедии (которая хочет знать ваше мнение, что же она для вас сделала за эти 15 лет)
Аноним 24/01/16 Вск 03:23:41 #390 №253793 
>>253780
>Земля, как центр БВ
>охуительные истории
>полуебка
хуйню написал ты
>у тебя каша в голове
>Пиздец
>отъебись
Аноним 24/01/16 Вск 03:34:32 #391 №253795 
>>253781
>и да я чето поскромничал, надо было сразу ионов сказать, те плазмы
С плазмой другая хуйня, там обратный Комптон-эффект, то есть высокоэнергетические электроны сообщают энергию фотонам с меньшей энергией, получаются высокочастотные фотоны с большой энергий. Будет тебе мощное рентгеновское излучение, что наблюдается возле таких замечательных объектов, как черные дыры и нейтронные звезды. Что касается ядер, оторванных от электронов, то есть ионов по сути, то на них Комптон-эффект гораздо ниже. Более того попадание гамма-квантов с огромной энеригией в ядро атома, выбивает нуклоны из ядра, что называется это гигантским дипольным резонансом. Мало того, разгоняя частицы в магнитных полях ты вызываешь синхротронное излучение, то есть как раз такое, как в аккреционных дисках нейтронных звезд и черных дыр, о чем я писал выше.

>фотосфера солнца
Это все равно, что прикрываться просто солнцем от гамма-всплеска. Это только в том случае, если Солнце на самом деле удачно встанет между нами. Кстати, гамма-всплеск отлично "дунет" солнечным говном в нашу сторону, не факт, что нам полегчает от этого.
>сколько в граммах
Столько не унесешь. я как раз недавно считал, >>247806 извини, что в килотоннах, а не в граммах.

>но ты реально хочешь пободаться вокруг этого изначально тупого вопроса
Нет, не хочу, я просто указал, что магнитным полем гамма-излучение невозможно отклонить , это был саркастический ответ на тот вопрос. Потом пошло про Комптон-эффект, который все равно не поможет.

>>253793
>хуйню написал ты
Я только процитировал очевидный бред, высранный тем полуебком.
И я же просил отъебаться от меня.
Аноним 24/01/16 Вск 03:50:09 #392 №253801 
>>253795
>Я только процитировал очевидный бред, высранный тем полуебком.
Давай посмотрим, что ты там процитировал...

>Земля, как центр БВ, который произошел локально, уже в некоей существующей Вселенной
Кто, окромя тебя, сказал, что Земля - центр БВ?
Откуда взялись слова "локально", и "существующей"?
Разве это цитата?

Каким
>полуебком
>высранный
этот
>бред
?

Теперь, иди туда, куда был послан прежде, и смотри видос. Если читать не умеешь.
Аноним 24/01/16 Вск 04:08:04 #393 №253802 
Когда люди поняли что другие планеты СС похожи на Землю, что существует принципиальная разница между звёздами и планетами помимо движения на небе?
Аноним 24/01/16 Вск 04:51:50 #394 №253807 
14536003102040.jpg
1) Точки Лагранжа описывают положение тела на орбите. Но он только описывают явление, а не задают его. Почему именно 1 тело на орбиту? Почему не может быть 2, 3, 4, 6 тел на орбите? Вон, VY Большого Пса такая толстая, что там можно на одну обиту разместить миллион Земель подряд. Или например разместим 4 Земли на нышней земной орбите вокруг солнышка стабильно. Ведь возможно же, просто будет более сложно объектам на влёт/вылет с Земли из-за того, что гравиколодец будет глубже. Ведь так?
2) Почему если бы на Марсе была человекоподобная жизнь, то люди там обязаны бы быть там выше типа из-за низкой гравитации? Что за бред? Почему они не могут быть наоборот меньше, чтобы соответствовать планете? Типа, универсальная пропроция челосущества к радиус(гравитации) планеты?
Аноним 24/01/16 Вск 04:57:58 #395 №253808 
>>253807
Голубь сдох.
http://www.youtube.com/watch?v=8UjESRXzEq8
Аноним 24/01/16 Вск 05:06:38 #396 №253811 
>>251606
> Что за быдло в треде. Пиздец. Перебанить бы вас всех, рачьё.
Интелектуал закукарекал!
У нас свободная борда, где любое мнение имеет право на жизнь.

Спасибо, Абу!

Абу благословил этот пост!
Аноним 24/01/16 Вск 05:07:41 #397 №253812 
>>253807
>Или например разместим 4 Земли на нышней земной орбите вокруг солнышка стабильно.
Не выйдет - объект как земля по массе исказит картину и точки лагранжа будут другие. Если объект небольшой по массе, такой чтобы считать его влияние на систему двух тел минимально, типа спутника или астероида, то можно сколько угодно.
Аноним 24/01/16 Вск 05:20:13 #398 №253813 
14536020135360.jpg
В будущем мы научимся переносить своё сознание в звезды
Аноним 24/01/16 Вск 05:57:19 #399 №253817 
>>253802
Отдельные товарищи догадывались еще в древности, именно из-за разницы движения по небу. В Европе начиная с 15 века примерно начались предположения. А дальше с развитием оптической астрономии. При изменения кратности звезды, в отличии от планет не увеличивались в окулярах.
>>253807

>Почему именно 1 тело на орбиту?
Потому что два тела на орбите представляют из себя довольно неустойчивую систему, но небольшое время могут существовать на этапе формирования планетарных систем. На орбите Земли, как считается, существовала Тейя, закончилось все столкновением. В точках Лагранжа обычно находят группы троянских астероидов, которые там болтаются и ни во что крупнее "слипнуться" не в состоянии.

>>253807
Скажем, им было бы проще на Марсе вырасти до таких размеров, например киты - огромны, но они живут в воде, которая помогает им справляться со своим весом. Но в первую очередь, на размер существ влияют условия существования. Климат, доступность пищи, влажность, средняя температура, наличие хищников и многое иное, и эти факторы гораздо важнее. Таким образом, при всех равных условиях, кроме гравитации, животным проще было бы вырасти большими: им бы не требовалось очень большое и сильное сердце, что бы качать кровь вверх, например, их кости не должны были бы быть слишком твердыми и массивными, что бы выдерживать вес, их мышцы не должны быть такими сильными. Что до размеров планеты, то большой ареал обитания нужен только либо суперхищникам больших размеров, либо животным, которые существуют очень большими стадами, так как им нужна большая питательная база. Что касается размеров планеты, то целый прайд львов контролирует всего ишь несколько десятков квадратных километров. То есть весь Марс можно было бы заселить полутора миллионами прайдами львов. Полмиллиона слонов пасутся на 5 миллионах квадратных километрах. Площадь Марса 144 800 000 км².
Аноним 24/01/16 Вск 07:02:02 #400 №253823 
нужно срочно знать, что происходит с гравитацией массы, если она превращается в энергию?
Аноним 24/01/16 Вск 12:03:15 #401 №253842 
>>253813
А трахаттса как???
Аноним 24/01/16 Вск 13:34:29 #402 №253847 
>>253842
Через протуберанцы и корональные дыры
Аноним 24/01/16 Вск 14:33:53 #403 №253855 
>>253817
> Таким образом, при всех равных условиях, кроме гравитации, животным проще было бы вырасти большими: им бы не требовалось очень большое и сильное сердце, что бы качать кровь вверх, например, их кости не должны были бы быть слишком твердыми и массивными, что бы выдерживать вес, их мышцы не должны быть такими сильными.
Охренительная логика. Тогда получается, жители Венеры, если бы она сейчас была как Земля, обязаны бы были быть меньше среднестатистического человека, ведь Венера малясь потяжелее. А если бы Юпитер был твёрдым, то там вообще бы жили коротышки с Незнайкой в Солнечном городе, так?
Это не логино.
Аноним 24/01/16 Вск 14:36:48 #404 №253857 
>>253795
>Столько не унесешь. я как раз недавно считал, >>247806 извини, что в килотоннах, а не в граммах.
считал он, помню я помню этот высер
что ты там посчитал, так это перевод эргов в джоули.
с темже успехом я могу стращать струей мюонов, естественно с дикими потоками по мощности, вылетающих из открывшихся рядом с солнцем порталов, и прям таки форсирующие наше солнышко порядка на 4ре..
Или поспекулировать о том как плохие парни анигилируют квазарчик, и в нашей галактике вообще не останется места человеку.
Или давай поспекулируем как этот луч попадает точнёхенько в земелюшку нашу русскую и все обитатети земли(в виде плазмы) ускоряются до второй световой.

Где какието другие присущие вспышке параметры, время, зависимость интенсивности, откуда она может взяться, спектр.
Но наверно это все было выше и я просто пропустил, ну ок бывает, я тут не все читаю.

>Нет, не хочу, я просто указал, что магнитным полем гамма-излучение невозможно отклонить , это был саркастический ответ на тот вопрос
где ответ в твоем сарказме, это раз.
второе, я бы не стал утверждать, что даже просто сферическая система в вакууме, в виде неоднородного магнитного поле и система гаммаквантов различных энергий - будет описываться словосочетанием - ничего не происходит. И уж точно не стал бы этого делать, для реальных систем.
но я хуй простой и многого не знаю, такчто бейте не бейте мне пох

и чето меня на последок зацепило
>Это все равно, что прикрываться просто солнцем от гамма-всплеска.
>просто солнцем от гамма-всплеска
>просто солнцем
Аноним 24/01/16 Вск 14:39:08 #405 №253859 
>>253842
Через аккрецию.
Аноним 24/01/16 Вск 14:45:21 #406 №253862 
>>248666 (OP)
как в невесомости происходит процесс пищеварения, почему содержимое желудка не вылетает обратно, ну ты понял?
Аноним 24/01/16 Вск 14:50:29 #407 №253867 
>>253795
>то есть высокоэнергетические электроны сообщают энергию фотонам с меньшей энергией, получаются высокочастотные фотоны с большой энергий.
плазма с температурой в пару электронвольт это тоже плазма, а не релятивистские электроны служащие умножителем, как ты пишешь
Аноним 24/01/16 Вск 15:02:39 #408 №253876 
>>253855
венера на 1/5 легче земли, ускорение свободного падения 0.9 земного
Аноним 24/01/16 Вск 15:05:00 #409 №253878 
>>248666 (OP)
Я ей нравлюсь?
Аноним 24/01/16 Вск 15:12:14 #410 №253882 
>>253862
перистальтика кишечника, в отличии от простейшей гидры, система пищеварения человека достаточно сложный механизм.
Даже вися вниз головой на турнике, процесс переваривания не останавливается
Аноним 24/01/16 Вск 16:34:02 #411 №253900 
>>253855
Я нигде не писал, должны быть, я написал, что при всех прочих равных им было бы проще быть больше. Хотя бы чисто энергетически. Не надо пытаться свою шизофрению подселять в мою голову.

>>253857
>я могу стращать струей мюонов
Стращай, если можешь. Но свои влажные фантазии лучше излагай в сайфай, или врачу. Какие-то пришельцы, порталы, квазары. К чему весь этот поток сознания?
>Где какието другие присущие вспышке параметры
Если ты вообще слыхом не слыхивал про гамма-всплески и их параметры, нахуй вообще лезть, что за мода?

>>253867
Электронвольт-это энергия, а не температура, болезный.

У вас обострение что ли? Погода поменялось, или вспышка на солнце?
Аноним 24/01/16 Вск 17:34:39 #412 №253917 
>>253882
Да всё проще: есть мышцА, "запирающая" пищевод
https://ru.wikipedia.org/wiki/Нижний_пищеводный_сфинктер
Обезьяна сконструирована так, что как ее ни болтай, ни говно из жопы, ни еда из желудка, не вываливаются. Запаяна с обоих концов.
Аноним 24/01/16 Вск 19:55:40 #413 №253946 
14536545410310.jpg
>>253372
>Может быть, я представил 4мерную сферу?
Ребят, я походу вообразил - сферу Паскаля.

Паскаль говорил, что Вселенная - это шар, поверхность которого нигде, а центр везде.
Такие дела.

>>252969>>252984>>253218>>253233>>253593>>253719-кун
Аноним 24/01/16 Вск 20:00:41 #414 №253950 
14536548412770.gif
14536548412771.gif
14536548412772.gif
>>253659
>>253719
>>253729
Просто оставлю это здесь.
Аноним 24/01/16 Вск 21:23:03 #415 №253993 
>>253900
>Где какието другие присущие вспышке параметры
>Если ты вообще слыхом не слыхивал про гамма-всплески и их параметры, нахуй вообще лезть, что за мода?
давай ты не будешь воображать себя здесь самым умным, есть вопрос - где все остальные параметры твоей пшишки.

>Электронвольт-это энергия, а не температура
хотя ладно проехали.
Аноним 25/01/16 Пнд 01:22:37 #416 №254088 
14536741575110.png
Ситуация: Солнце мгновенно и полностью закрыли и с Земли его не видно. Для наблюдателей с Земли, Солнце исчезнет через 8 минут?
Аноним 25/01/16 Пнд 02:17:58 #417 №254109 
>>254088
Да
Аноним 25/01/16 Пнд 04:59:18 #418 №254148 
>>254109
Нет
Аноним 25/01/16 Пнд 05:23:01 #419 №254154 
>>254148
Пидора ответ.
Аноним 25/01/16 Пнд 08:27:58 #420 №254200 
А есть какая-нибудь статистика по поводу иноземной жизни? Какова вероятность ее возникновения?
Аноним 25/01/16 Пнд 08:52:03 #421 №254204 
>>254200
Вероятность отлична от нуля. Но жизни пока не обнаружено.
Аноним 25/01/16 Пнд 09:02:34 #422 №254205 
>>252580
Есть, но платные.
Аноним 25/01/16 Пнд 10:58:52 #423 №254219 
14537087325460.png
Если в "вакууме" "бросить" на землю объект с массой как у пера и с массой как у солнца, то они будут падать с разной скоростью?
Аноним 25/01/16 Пнд 11:02:03 #424 №254220 
>>254219
Да
Аноним 25/01/16 Пнд 11:05:03 #425 №254221 
>>254220
А есть пруфы? Статья на вики про зависимость массы и скорости притяжения, например?
Аноним 25/01/16 Пнд 11:08:01 #426 №254223 
>>254219
Ты опять выходишь на связь?
Аноним 25/01/16 Пнд 11:09:56 #427 №254224 
Посоветуйте игорь на ведроид. Графоний не особо интересует.
Вот помню игра была, в ней нужно было приземлиться на луну попердывая из сопел.
Аноним 25/01/16 Пнд 11:14:17 #428 №254225 
>>254223
Нет. Просто мне это утверждение кажется голословным, нужны аргументы.
Аноним 25/01/16 Пнд 11:14:57 #429 №254226 
>>249164
Это не пыль, это копоть от ракет на хрустальный свод налипла.
Аноним 25/01/16 Пнд 11:18:33 #430 №254227 
>>254225
Тебе выше по треду все с формулами пояснили.
Аноним 25/01/16 Пнд 11:21:59 #431 №254229 
>>254227
Значит это был не я. Прочел тред, нету тут такого
Аноним 25/01/16 Пнд 11:24:56 #432 №254231 
>>254229
А, это в прошлом было. Вот тебе начало ветки, отсюда читай: >>247546
Аноним 25/01/16 Пнд 11:32:28 #433 №254232 
>>254231
Спасибо
Аноним 25/01/16 Пнд 11:32:32 #434 №254233 
>>253258
>Флаг на Луне развивается
Далеко развился? Скоро земляшку покорит?
Аноним 25/01/16 Пнд 12:04:05 #435 №254238 
>>254233
Да, скоро уже прилетят текстильные звёздно-полосатые прогрессоры, вытаскивать земляшку из каменного века
Аноним 25/01/16 Пнд 12:05:37 #436 №254239 
14537127373320.jpg
Анон, подкинь годных документалок на тему изучения космоса. Пока что смотрел только очевидную "spacetime odyssey" с Тайсоном, и очень доставило. Хотелось бы чего-то настолько же годного в плане подачи, но более конкретизированного — особенно про историю космонавтики и изучение Солнечной Системы и её объектов.
Аноним 25/01/16 Пнд 12:15:37 #437 №254241 
>>253878
Нет. Мне тоже не нравишься, уйди
Аноним 25/01/16 Пнд 12:33:52 #438 №254247 
>>254239
http://rutracker.org/forum/viewtopic.php?t=4510772 - более-менее близко к реальности показаны полеты амерских астронавтов
http://rutracker.org/forum/viewtopic.php?t=5138785 - то же самое, но для баб, смотри не блевани от соплей
Аноним 25/01/16 Пнд 13:22:23 #439 №254256 
>>254221
>А есть пруфы?
ну, законы Ньютона там, например
Аноним 25/01/16 Пнд 13:59:09 #440 №254259 
Поясните вот что:
Где-то слышал, что одной из главных проблем полета человека к Марсу является высокий уровень радиации. Что мол, к моменту прилета на Марс, человек уже получит почти смертельную дозу (ну или очень высокую, не помню), это если не брать в расчет вспышек на Солнце (которые, за время полета, почти 100% будут). И что для решения этой проблемы, нужно оснащать корабль нихуевой защитой, которую доставить на орбиту почти не реально. Так вот, какого черта сейчас наса кукарекают о том как они собираются все-таки запустить туда людей. Причина банально в попытке привлечь средств? Или реально существуют пути решения?
Аноним 25/01/16 Пнд 14:03:44 #441 №254261 
>>254259
Гагарин летал - бога не видел Куриосити летал - радиации не нашел. За полет до Марса намерил уровень радиации в 2 раза больше чем на МКС, что неприятно, но не смертельно. Проблема вроде как лишь мощные солнечные вспышки.
Аноним 25/01/16 Пнд 14:43:54 #442 №254268 
>>254261
Марс вообще-то активно подвергается бомбандировке нейтронами
Аноним 25/01/16 Пнд 14:46:45 #443 №254270 
>>254233
>развивается
>Далеко развился?
Развевается, да. fix
Аноним 25/01/16 Пнд 14:53:27 #444 №254271 
>>254268
>активно подвергается бомбандировке
http://geektimes.ru/post/262252/
Аноним 25/01/16 Пнд 14:57:59 #445 №254273 
14537230795590.jpg
>>254259
В результате миссии, которая будет состоять из 180 дней путешествия в один конец (к Красной Планете или обратно к Земле) и 500 дней, проведенных собственно на Марсе, человек получит суммарную дозу облучения, равную 1.01 зиверта, - таков результат измерений, проведенных детектором излучений ровера Radiation Assessment Detector (RAD).

Европейское Космическое Агентство ограничило допустимую дозу облучения, которую получают космонавты за все время своей работы, 1 зивертом – при этом риск возникновения злокачественных опухолей возрастает на 5%.

Данные RAD показывают, что космонавты, исследующие поверхность Марса, будут получать дозу, равную приблизительно 0.64 миллизиверта каждый день.
Во время путешествия к Марсу уровень радиации будет выше приблизительно в три раза - 1.84 миллизиверта каждый день.

Такие дела.

http://www.astronews.ru/cgi-bin/mng.cgi?news=5106&page=news
Аноним 25/01/16 Пнд 15:10:51 #446 №254276 
>>254273
Опасная хуйня этот ваш космос.
Аноним 25/01/16 Пнд 15:11:31 #447 №254277 
>>254273
Бля, солнце же сраные фотоны раскидывает, без массы без нихуя и само из гелия с водородом состоит, откуда блядь там радиация нахуй?
Аноним 25/01/16 Пнд 15:13:34 #448 №254279 
>>254271
Ну да блядь, на планету без атмосферы микробов заселить, как же.
Аноним 25/01/16 Пнд 15:20:46 #449 №254282 
>>254271
Радиация в капсуле на марсе в 2 раза больше чем на мкс. Пока долетим уже облучимся, долетим получим очень тяжелую дозу. Будем выходить на поверхность в светлое время суток получим смертельную.
Аноним 25/01/16 Пнд 15:21:56 #450 №254283 
>>254271
Погугли что такое нейтрон. Алсо новость баян
Аноним 25/01/16 Пнд 15:30:17 #451 №254285 
>>254277
Дейтерий плюс тритий равно ядро гелия с нейтроном
Аноним 25/01/16 Пнд 15:34:35 #452 №254286 
2 зиверта летальны? Тут полторашка.
Аноним 25/01/16 Пнд 15:42:27 #453 №254289 
>>254286
суточная норма же.
Аноним 25/01/16 Пнд 15:47:46 #454 №254291 
>>254289
Может микрозиверта?
Аноним 25/01/16 Пнд 16:03:55 #455 №254293 
>>254286
Если схватить за короткий промежуток времени, можно и от одного зиверта откинуться с некоторой вероятностью. С радиацией вообще все непросто, в некоторых ситуациях можно и сотню зивертов получить и не откинуться.
Аноним 25/01/16 Пнд 16:03:58 #456 №254294 
>>254286
>>254289
>>254291
Зиверты, вроде, копятся.

>>254282
Есть такая цаца, как высокотемпературная сверхпроводниковая тороидальная магнитная система типа «рейстрэк» из 24 обмоток, может помочь в освоении дальнего космоса.

http://perst.isssph.kiae.ru/supercond/bulletein.php?menu=bull_subj&id=1175
Аноним 25/01/16 Пнд 16:17:20 #457 №254300 
>>254294
Если она генерит магнитное поле, а не высокотемпературную плазму то збс.
Можно будет спокойно укрываться внутри капсул, и периодически устраивать вылазки.
Аноним 25/01/16 Пнд 16:46:16 #458 №254307 
Что если солнце как чернобыль в бетонный короб закрыть?
Аноним 25/01/16 Пнд 16:49:21 #459 №254309 
Я - дрищ. Если я заберусь в скафандр, и ещё на орбите прицеплю к себе уже открытый парашют от БМД доблестных ВеДеВе, смогу ли я задеорбититься целым и живым, пусть даже несколькими заходами в атмосферу на пол-шишечки?
Аноним 25/01/16 Пнд 16:51:57 #460 №254312 
>>254309
Нет, всё равно сгоришь/распидорасит перегрузками.
Аноним 25/01/16 Пнд 16:55:14 #461 №254313 
>>254312
Что-то мне подсказывает, что ты спиздел просто так.
Парашют - охуенная площадь, дрищавость - охуенно малый вес.
Вон, от рузге разгонных блоков куски долетают аж до Вьетнама, и норм, а там отношение площади к весу ещё меньше. Стеклопластиковые конструкции от Скулабе тоже целыми и даже не обгоревшими долетали.
Аноним 25/01/16 Пнд 16:56:15 #462 №254314 
Наноботы. Делаешь себе инъекцию и радиация тебя не берет.
Аноним 25/01/16 Пнд 16:58:23 #463 №254317 
>>254313
Дебил блядь, скорость падения мкс посмотри, приблизительно с такой же скоростью и ты будешь падать с поправкой на твой вес.
Аноним 25/01/16 Пнд 16:58:43 #464 №254318 
14537303231550.png
Почему VASIMR не взлетает. Идея же шикарная. Помещаешь рабочее тело в магнитную ловушку, разогреваешь его до 1*10^10000 хуйлиардов градусов и выпускаешь наружу из магнитного сопла. Ящитаю это гениальная недооценённая идея. Ведь тут и экономия массы рабочего тела и что куда важнее - вся суть в удельном импульсе. Тоесть надо не большую массу фигово нагреть, а маленькую массу нагреть так чтобы огого. Ведь если всю аппаратуру (аккумулятор-ионистр-конденсатор-кабеля-катушки) сделать из толковых сверхпроводников, то получится лёгкий и мощный движок. Объясните в чём косяк VASIMRа
Аноним 25/01/16 Пнд 17:00:59 #465 №254319 
14537304596040.webm
>>254318
>329x153
Аноним 25/01/16 Пнд 17:08:05 #466 №254323 
Два вопроса.

1) Гравитационное двигло возможно?
2) Шо там с тремя огромными кораблями лунатиков? Слышно, что или их уже "сбили"?
Аноним 25/01/16 Пнд 17:12:45 #467 №254325 
14537311652350.jpg
>>254319
Аноним 25/01/16 Пнд 17:15:31 #468 №254328 
Не верю.
Аноним 25/01/16 Пнд 17:17:56 #469 №254330 
>>254314
Предлагаешь, на иглу садиться?
Что там с дозировкой этих наноботов?
Аноним 25/01/16 Пнд 17:20:03 #470 №254332 
>>254323
Ты и так на гравитационном двигде живешь.
Аноним 25/01/16 Пнд 17:21:46 #471 №254333 
>>254332
Я про что-то более компактное и применимое для полёта. Я делаю игру, но мне что-то влом делать, слишком "твёрдые" корабли. Но хочу более-менее реалистично. >_<.
Аноним 25/01/16 Пнд 17:24:59 #472 №254336 
>>254330
Неправильно поставлен вопрос.
>Что там с существованием и принципом работы этих наноботов?
А то обыграются в свой круйзис, и колють наноботов в вену по подъездам.
Аноним 25/01/16 Пнд 17:41:36 #473 №254341 
>>254314
Да ты что блядь! Сравни размеры элементарной частицы и нанобота. И в чем суть нанобота? Он будет чинить разьебаную клетку или сломанную днк? Это невозможно
Аноним 25/01/16 Пнд 18:15:01 #474 №254356 
Как собрать поле Геллера из подручных средств?
Аноним 25/01/16 Пнд 18:15:36 #475 №254358 
>>254341
Все равно будет больше твоего пиструна.
Аноним 25/01/16 Пнд 18:28:55 #476 №254364 
14537357359990.jpg
>>254356
Вахач в другой стороне, родной.
Аноним 25/01/16 Пнд 18:40:11 #477 №254366 
Можно ли каптчевать на МКСе? Достанет ли до туда юсби модем?
Аноним 25/01/16 Пнд 20:02:15 #478 №254388 
>>254366
А они по твоему телепатически общаются?
Аноним 25/01/16 Пнд 20:37:01 #479 №254402 
>>254318
В охлаждении? На солнышке-то, сверхпроводники по пизде не пойдут?
Аноним 25/01/16 Пнд 20:40:32 #480 №254406 
>>254307
Он сколлапсирует и упадёт на солнце, после чего светило будет нное время срать бурями и гигантскими протуберанцами
Аноним 25/01/16 Пнд 20:55:26 #481 №254411 
>>254277
>сраные фотоны
/this
плюс там сам по себе фон от всягого говна
Аноним 25/01/16 Пнд 20:55:46 #482 №254412 
>>254402
Наоборот бесплатный обогрев.
Аноним 25/01/16 Пнд 21:02:00 #483 №254413 
>>254289
фон гдето 0.08-0.180 mkSv/час те примерно (умножаем на 24 0.1 среднее по больнице) 2-3 mkSv сутки, в год у тебя милизиверт получится. Это фон на земле. На мкс на три порядка выше.
Вообщем если репродукция все еще интересует, то не рекомедуется, просто во избежание. Для работников соотвествующих, там нормы сещественно выше фона, но не помню сейчас сколько.
Аноним 25/01/16 Пнд 21:10:15 #484 №254414 
14537454157880.jpg
>>254336
>круйзис
Crysis*. fix.
Это классика - это знать надо!
Аноним 25/01/16 Пнд 21:41:53 #485 №254416 
>>254309
с текущим оборудованием нет

>>254318
затрату по энергии растут пропорционально квадрату роста импульса
в качестве ускорителя используется потенциал между электродами, и соотвественно тот электрод который на выходе накернивается плазмой, нагревается и идет по пизде.
превращение вещества в плазму тоже достаточно затратный процесс
потенциалы ионизации http://www.chemway.ru/bd_chem/tbl_mol/w_tbl_r_m_02.php
есть теория такая хуита на йоде
http://www.energia.ru/ktt/archive/2013/02-2013/02-04.pdf
ксенон юзают, потому что энергия на ионизацию как на водород, а разгоняемая масса в 130 раз больше чем у водорода (те грубо для манипуляции с водородом пришлось бы выкинуть в 130 раз больше энергии, для тех же по сути результатов)
ксенон тоже не халявный
плотность плазмы не высокая, те тяга на сопло тоже не ахти

Основной косяк это энергия.
Так сказать активация топлива(ионизация) 1эВ это значит надо потратить 100кДж на один моль реактивной массы - просто чтобы начать его разгонять.(что могло бы соответствовать УИ 5200с для водорода, 445с для ксенона (если я не сильно наебался))
Аноним 25/01/16 Пнд 21:45:23 #486 №254417 
>>254341
он может не давать распространятся метастазам и накернивать источник метастаз
потеря единичных клеток для человека не существенна.
Аноним 26/01/16 Втр 06:09:25 #487 №254517 
>>254417
Лол.
Аноним 26/01/16 Втр 06:12:12 #488 №254518 
>>254406
С чего бы это ему коллапсить?
Аноним 26/01/16 Втр 06:14:57 #489 №254520 
К тому времени когда солнце начнет пидорасить, мы усилим земное магнитное поле настолько что солнечные ошметки будут пролетать 10 дорогой не задавая землю.
Наверное это было бы охуенно, наблюдать как на нас летит плазма, а потом внезапно словно ее кто-то отталкивает в другую сторону и мы спасены.
Аноним 26/01/16 Втр 07:30:58 #490 №254526 
>>254518
Пушто ты не разогнал его до орбитальной скорости.
Аноним 26/01/16 Втр 07:32:43 #491 №254527 
>>254520
Это кино будет называться "Горизонт событий в ручную"?
Аноним 26/01/16 Втр 08:53:50 #492 №254535 
14537876301630.jpg
На Марсе будет работать пикрелейтед? Низкая гравитация и песок вроде должны работать на идею воздушной подушки.
Аноним 26/01/16 Втр 08:55:16 #493 №254536 
14537877168460.jpg
Сможет джетпак поднять вес небольшого скафандра на Марсе?
Аноним 26/01/16 Втр 09:10:23 #494 №254537 
Сколько стоит и где купить марсоход?
Аноним 26/01/16 Втр 09:11:15 #495 №254538 
>>254535
Там очень разряженная атмосфера. Заебешься нагонять давление в подушке.
Аноним 26/01/16 Втр 09:21:25 #496 №254539 
>>254536
Не слишком тяжелого - да.
>>254537
2,5 миллиарда долларов стоит Кьюриосити
Заказать можешь в Boeing/Lockheed Martin. Поскольку проект уже есть, и если тебе не не нужны натурные дубли на Земле, то очевидно, дадут большую скидочку.
И не забудь отложить на операционные расходы.
Аноним 26/01/16 Втр 09:32:27 #497 №254541 
>>254527
И порно-версию запилите плес. Хотеть на это подрочить.
Аноним 26/01/16 Втр 09:34:57 #498 №254542 
>>254539
Чет дорого. Пойду лучше в компанию РадиотехЗапчасти на базаре и куплю нужные запчасти.
Аноним 26/01/16 Втр 09:37:39 #499 №254544 
Какое уравнение решал мистер Бренд в interstellar'e?
Чому нигер сказал что если мы увидим внутри гаргантюши сингулярность или коллапсар то мы обуздаем гравитацию?
Аноним 26/01/16 Втр 09:40:36 #500 №254545 
14537904368720.jpg
Вот это хард. Неужели он не коцается?
Аноним 26/01/16 Втр 09:47:57 #501 №254546 
>>254545
Ты же знаешь что нет
Аноним 26/01/16 Втр 10:01:39 #502 №254547 
>>254545
А как наши экзомарс планируют сажать? Или они не планируют и не собираются?
Аноним 26/01/16 Втр 10:28:51 #503 №254553 
>>254547
Планируют. Не собираются.
Аноним 26/01/16 Втр 10:30:38 #504 №254555 
>>254544
Смысл различных математических символов, входящих в уравнение профессора, раскрыт на остальных пятнадцати досках. Уравнение выражает «действие» S (классический предел «квантового эффективного действия») в виде интеграла лагранжевых функций L. Эти функции связаны с геометрическими свойствами пространства — времени («метриками») пятимерного балка и нашей четырехмерной браны, с набором полей, действующих в балке (которые обозначены как Q, σ, λ, ξ и φi), а также с «полями стандартной модели», действующими в нашей бране (включая электрические и магнитные поля). Поля и пространственно-временные́ метрики варьируются, чтобы найти экстремум (максимум, минимум или седловую точку) действия S. Условия экстремума представляют собой систему уравнений Эйлера — Лагранжа, которые определяют эволюции полей; это стандартная процедура вариационного исчисления. Профессор и Мёрф делают предположения относительно полей балка φi, неизвестных функций U(Q), Hij(Q2), M (поля стандартной модели) а также неизвестных констант Wij, которые входят в лагранжевы функции. На рис. 25.8 я записываю на доске список этих предпо- ложений. Затем для каждого набора предположений профессор и Мёрф варьируют поля и метрики пространства — времени, вы- водят уравнения Эйлера — Лагранжа, а затем выполняют компьютерное моделирование, исследуя прогнозы этих уравнений относительно гравитационных аномалий.

Если мы возьмем обычные квантовые законы, не учитывающие гравитацию, а затем отбросим флуктуации, мы получим законы ньютоновской физики — законы, которые в течение нескольких последних столетий использовались для описания планет, звезд, мостов и бильярдных шаров. Если же взять законы квантовой гравитации (о которых мы знаем пока немного) и пренебречь флуктуациями, то должны получиться законы теории относительности (которые изучены куда лучше). Флуктуации, которыми мы пренебрежем, — это, например, пена из крохотных флуктуирующих червоточин («квантовая пена», которой пронизано все пространств. Без учета флуктуаций законы теории относительности точно описывают искривление пространства и времени вблизи черной дыры и замедление времени на Земле. Если бы профессору Брэнду удалось открыть законы квантовой гравитации и для балка, и для нашей браны, тогда, исключив из этих законов флуктуации, он мог бы найти точную форму своего уравнения. И узнал бы причину гравитационных аномалий и как ими можно управлять — то есть как можно их использовать для эвакуации человечества с Земли. Сингулярности — это розеттский камень для расшифровки квантовой гравитации.
Аноним 26/01/16 Втр 10:40:02 #505 №254558 
>>254547
>А как наши экзомарс планируют сажать?
Наши не собираются. Наши предоставляют протон и Бриз-М. Европейца будут сажать по-старинке и без выебонов: тепловой щит и парашютная система, в самом конце посадки будут задействованы тормозные двигатели.
Аноним 26/01/16 Втр 10:40:12 #506 №254559 
Я люблю тебя, спейсач
Аноним 26/01/16 Втр 10:49:26 #507 №254563 
14537945662240.jpg
>>254558
В чем разница между lander и landing platform? Вроде пишут что что-то будет Russian.
Аноним 26/01/16 Втр 10:53:30 #508 №254565 
>>254555
Ну увидел ты сингулярность, а дальше что?
Аноним 26/01/16 Втр 11:06:50 #509 №254569 
>>254563
Да, в самом деле, посадочная платформа будет наша. Что-то я этот момент пропустил.
Аноним 26/01/16 Втр 11:10:22 #510 №254570 
А я бы полетел на марс в один конец даже в одиночку. Но при условии что я смогу двачевать. с задержкой в 7 минут, ну да ладно.
Всё равно я хиккан.
Хотя пожалуй будет еще одно условие:возможность осуществления дальних вылазок.
А потом я внезапно наткнусь на необычную породу, передам данные на землю, на земле решат что порода состоит из ранее неизвестного химического элемента и теоретически может быть новым наименее затратным топливным элементом для всего.
Экономисты решат что освоение марса это большая выгода и пришлют туда кучу народу вместе с которой мы будем строить колонию и горнодобывающую станцию, воть.
Аноним 26/01/16 Втр 11:20:07 #511 №254572 
>>254565
>а дальше что?
Дальше TARS берет кайло и добывает науку. Че как маленький?
>>254570
>с задержкой в 7 минут
Для тебя плохие новости: 3 минуты сигнал идет в одну сторону только при максимальном сближении планет, а это бывает очень не часто.
Аноним 26/01/16 Втр 11:22:38 #512 №254574 
>>254572
Ну и ладно. Зато от осознания того что стоишь в хулиарде км от дома на другой плонети - сносит башню.
Аноним 26/01/16 Втр 11:43:26 #513 №254578 
14537978069500.jpg
14537978069601.jpg
>>254563
Там два компонента при посадке: лендер и ровер. Лендер (Descent Module/десантный модуль) останется на месте. На нем тоже есть инструменты (от ИКИ и ЕКА), это самостоятельный аппарат, который будет изучать долговременные изменения и климат. Ровер съедет с него и покатит по своим делам. Марсоход делается подрядчиками ЕКА, десантный модуль делают в Лавочкине на базе EDM - посадочного модуля Скиапарелли, по заказу ЕКА. (большая часть работы на Лавке и ИКИ, т.к. EDM довольно сильно отличается от того, что нужно для ровера).
Аноним 26/01/16 Втр 11:49:21 #514 №254580 
>>254553
анон по хардкору.
можно перекатываться потихоньку Аноним 26/01/16 Втр 11:54:59 #515 №254582 
https://2ch.hk/spc/res/254581.html
https://2ch.hk/spc/res/254581.html
Аноним 26/01/16 Втр 11:55:20 #516 №254583 
>>254565
дальше все.

>>254572
пинг большого значения не имеет, если речь не о чатрулете и подобном
Аноним 26/01/16 Втр 16:38:16 #517 №254634 
>>254582
Перекатился тебе за щеку
Аноним 26/01/16 Втр 22:41:23 #518 №254814 
14538372836510.png
>>254570
>порода состоит из ранее неизвестного химического элемента
Вут?
Аноним 27/01/16 Срд 02:11:10 #519 №254870 
14538498710730.jpg
>>248666 (OP)
Что будет с российской космической программой, если глава роскосмоса говорит, что денег нет, производство по пизде и устарело, а импортные комплектующие не поставляются?
Аноним 27/01/16 Срд 12:25:43 #520 №254947 
>>253855
>ведь Венера малясь потяжелее
Хотел было тебя обоссать, но уже поздно.
Аноним 27/01/16 Срд 15:04:05 #521 №254971 DELETED
>>254582
Бамплимит - 2к постов.
Аноним 27/01/16 Срд 19:31:28 #522 №255057 
14539122882250.jpg
>>254870
>Что будет с российской космической программой
Ей пиздец.
Аноним 28/01/16 Чтв 03:12:43 #523 №255310 
>>254870
Сначала надо принять эту программу. Пока есть только проект, не принятый в правительстве, потом смотреть, что там в этом проекте осталось. Но в целом ничего хорошего не видно. Статус одного из основных извозчиков сохраним, в лучшем случае. Было-то полтора годных проекта, но и они либо отложены, либо заморожены. Вероятно, будет еще хуже.
Аноним 31/01/16 Вск 12:50:58 #524 №256814 
14542338587210.jpg
>>255057
нет тут нужен релейтед Гиркина.
Аноним 31/01/16 Вск 12:54:38 #525 №256818 
>>255310
>Статус одного из основных извозчиков сохраним, в лучшем случае
Ничерта не сохраним. половина уже к Маску ушла. Оставшаяся половина уйдет когда Маск выкатит Falcom Heavy, и после очередных удачных посадок на платформу, будет еще больший демпинг чем раньше. На 2017 у Протона коммерческих запусков почти нет.
Аноним 31/01/16 Вск 13:31:50 #526 №256837 
>>256818
на 2016 коммерческих пусков всего три (экзоМарс совместный)
на 2017 пока что один.
Дела не очень.
Зато отечественный заказ не плохой.
на 2016 - семь
на 2017 - три
Что очень странно. Не хотим пускать с Ангары. Хотим жирать говно нюхать гептил и платить казахам за не их космодром. Да мерийности носителя пока нет, но можно же было сделать партию пробных ракет A5 на 10-12 пусков??? как раз обкатали бы.
comments powered by Disqus

Отзывы и предложения